l13-l14 engr ece 4243-6243 11292016 f. jain solar cells

94
441 L13-L14 ENGR_ECE 4243-6243 11292016 F. Jain Solar Cells Contents 6.1. Introduction...................................................................................................................................442 6.1.1 Losses and Conversion Efficiency……………………………….…………..…….……….442 6.1.2 Material selection in solar cells……………………………………………….…..…….…..443 6.1.3 Concentrated Solar Photovoltaics (CSP) and Tandem Multi-junction Cells ……443 6.2 Solar Spectrum and Air Mass m……………………………………………………….………445 6.3 Absorption of Photons in Semiconductors……………………………………………….……446 6.4 Photovoltaic Effect………………………………………………………………………………451 6.4.1 Qualitative explanation…………………………...…………………………………………453 6.4.2 V-I Equation: Equivalent Circuit Approach……………………………………...…………454 6.4.3 Open Circuit Voltage Voc, Maximum Power Point and Fill Factor (FF) ………………456 6.5 Conversion Efficiency and Losses………………………………………...……………………459 6.5.1 Series and Shunt resistances in the equivalent circuit……………………...……….………459 6.6 Solar Cell Materials and Technologies: Generation I…………………….……...………..…461 6.7 Solved Examples (Example 2-4)…………..……………………………….……….…………..465 6.8 Solar Cell Design…………………………………………………………………………...……469 6.8.1 Solar Cell Design for Air Mass m=1………………………………………………..………469 6.8.2 Solar Cell Design for AM 0 (outer space)……………………………………...………...…480 6.8.2.1 Bar Chart of Losses for AM0, m = 0...........................................................484 6.9 Fabrication and Simulation of Solar Cells………………………………………………...…..486 6.10 Tandem Solar Cells……………………………………………………………………...…….488 6.10.1 Tandem Solar Cell Simulation………………………………………...……………...…...488 6.10.2 Tandem Solar Cells using amorphous and crystalline cells……………...…………...…...490 6.11 Tandem, MEG, and Quantum Dot Solar Cells: Part-II……….………………….......…….494 6.11.1 Emerging new cell structures……………………………………………………...…...….494 6.11.1.1 Fundamentals of Optical Transitions (Excitonic and free carrier)………...……….494 6.11.1.2 Nanostructures in Solar Cells and Current Technologies ……………..496 6.11.2 Heterojunction with intrinsic thin film (HIT) cell…………………………………...….....498 6.11.3 Thin film CdSe and CuInSe2 cells………………………………………………...…..….499 6.11.4 Multi-junction or Tandem Solar Cell 3 rd Generation………………………………….…..501 6.11.5 Multiple Exciton Generation (MEG), IB cells, and Quantum Dot based solar cells..........504 6.11.6 Organic Photovoltaics (OPVs)……………………………………………………..…...…507 6.12 Problem set and Solutions for Solar Cell Design and Tandem Cells…………………….…514 6.13 Derivation of V-I equation in an n-p solar cell………………………………………………524 6.14 Solar cell summary equations…………………………………………………………....……531 6.15 References…………………………………………………………………………..……...…533

Upload: others

Post on 25-Jan-2022

4 views

Category:

Documents


0 download

TRANSCRIPT

441

L13-L14 ENGR_ECE 4243-6243 11292016 F. Jain

Solar Cells Contents

6.1. Introduction...................................................................................................................................442

6.1.1 Losses and Conversion Efficiency……………………………….…………..…….……….442

6.1.2 Material selection in solar cells……………………………………………….…..…….…..443

6.1.3 Concentrated Solar Photovoltaics (CSP) and Tandem Multi-junction Cells ……443

6.2 Solar Spectrum and Air Mass m……………………………………………………….………445

6.3 Absorption of Photons in Semiconductors……………………………………………….……446

6.4 Photovoltaic Effect………………………………………………………………………………451

6.4.1 Qualitative explanation…………………………...…………………………………………453

6.4.2 V-I Equation: Equivalent Circuit Approach……………………………………...…………454

6.4.3 Open Circuit Voltage Voc, Maximum Power Point and Fill Factor (FF) ………………456

6.5 Conversion Efficiency and Losses………………………………………...……………………459 6.5.1 Series and Shunt resistances in the equivalent circuit……………………...……….………459

6.6 Solar Cell Materials and Technologies: Generation I…………………….……...………..…461

6.7 Solved Examples (Example 2-4)…………..……………………………….……….…………..465

6.8 Solar Cell Design…………………………………………………………………………...……469

6.8.1 Solar Cell Design for Air Mass m=1………………………………………………..………469

6.8.2 Solar Cell Design for AM 0 (outer space)……………………………………...………...…480

6.8.2.1 Bar Chart of Losses for AM0, m = 0...........................................................484

6.9 Fabrication and Simulation of Solar Cells………………………………………………...…..486

6.10 Tandem Solar Cells……………………………………………………………………...…….488

6.10.1 Tandem Solar Cell Simulation………………………………………...……………...…...488

6.10.2 Tandem Solar Cells using amorphous and crystalline cells……………...…………...…...490

6.11 Tandem, MEG, and Quantum Dot Solar Cells: Part-II……….………………….......…….494

6.11.1 Emerging new cell structures……………………………………………………...…...….494

6.11.1.1 Fundamentals of Optical Transitions (Excitonic and free carrier)………...……….494

6.11.1.2 Nanostructures in Solar Cells and Current Technologies ……………..496

6.11.2 Heterojunction with intrinsic thin film (HIT) cell…………………………………...….....498

6.11.3 Thin film CdSe and CuInSe2 cells………………………………………………...…..….499

6.11.4 Multi-junction or Tandem Solar Cell 3rd Generation………………………………….…..501

6.11.5 Multiple Exciton Generation (MEG), IB cells, and Quantum Dot based solar cells..........504

6.11.6 Organic Photovoltaics (OPVs)……………………………………………………..…...…507

6.12 Problem set and Solutions for Solar Cell Design and Tandem Cells…………………….…514

6.13 Derivation of V-I equation in an n-p solar cell………………………………………………524

6.14 Solar cell summary equations…………………………………………………………....……531

6.15 References…………………………………………………………………………..……...…533

442

6.1. Introduction

Solar spectrum and power distribution in various spectral regimes is a function of path

length in earth's atmosphere. The received power on earth is measured in terms of air mass

m. While m is taken to be '0' for outer space, it has a value of '1' when sun is at zenith. The

value of m increases as the sunlight travels more distance in atmosphere particularly in the

morning at dusk, so m is very high under these conditions.

Photon absorption and generation of electron-hole pairs

Photons are generally absorbed in semiconductors when their energy is above the band

gap Eg. This involves generation of electron-hole pairs (EHPs). An absorbed photon

produces one EHP.

The separation of electrons and holes before they recombine requires the presences of

a barrier. The barrier could be a p-n junction or a Schottky interface (metal-

semiconductor, MS, junction) or a MIS (metal-thin insulator ~20A-semiconductor)

interface.

The current flows from a solar cell to an external load. The maximum power transfer

point (Vm, Im) is defined to be at which the cell is delivering maximum power to the

load. The fill factor FF is defined as Vm Im /Voc Isc.

The conversion efficiency ηc (Vm Im/Pin) of a solar cell is determined by various losses.

6.1.1 Losses and Conversion Efficiency: These losses include

1. Surface reflection from the front surface.

Remedy: This can be reduced by having an antireflection (AR) coating.

2. Transmission of radiation which is below the energy gap Eg,

3. Excess energy (hυ –Eg) per photon that is not utilized in the formation of an electron-

hole pair, and is lost as heat.

Remedy: This loss can be minimized by having multi-junction or tandem cells. In these

cells photons from a spectral regime are absorbed in a specific layer. Various layers

are needed to absorb the full solar spectral range. Various regions are connected via

tunnel junctions. These cells are invariable heterojunctions.

4. Fill factor FF (the cell not exhibiting an ideal rectangular I-V characteristic).

Remedy: A cell should be fabricated with very small value of Rs, series resistance, and

a very high shunt resistance Rsh (see items 6 and 7).

5. Voltage factor (qVoc/Eg); the material can produce Eg/q terminal voltage but produces

only Voc.

Remedy: Design the cell such as Vbi is as high (and close to Eg/q) as possible.

6. Series resistance Rs (due to bulk material Ohmic resistance and the resistance of the

contacts).

443

7. Shunt resistance Rsh representing leakage of photo-generated current that does not pass

through the junction, instead recombines at the surface.

Remedy: Passivate the region around the junction so that there is no leakage current via

the surface and interface states.

8. Collection of generated electron-hole pairs; some carriers recombine before they are

separated by the junction.

Remedy: Make the contact spacings such that all regions of cells where electron and

hole pairs are created are within one diffusion length.

9. Absorption in the window region; carriers generated in this region generally recombine

in homojunctions before being they are separated. As a result the window region is

made as thin as possible. There is a tradeoff between Ohmic resistance and photon loss.

That is, we cannot make it too thin.

Remedy: Use heterojunctions. That is, make the window region of a wider energy gap

material.

6.1.2 Material selection in solar cells:

The maximum power that a cell can deliver is Isc *Voc. Isc depends on the band gap Eg;

it is high for smaller gap semiconductors, and Voc [~ kT/q * ln(Isc/Is)] depends on the ratio of

Isc and reverse saturation current Is. and Isc or IL. Since the reverse saturation current decreases

as we increase the band gap Eg, it is rather low for higher energy gap materials. As a result the

product of Isc *Voc peaks around Eg =1.4 eV.

The material could be single crystalline, polycrystalline, or amorphous. Generally the

materials fall in two categories: (1). high efficiency (over 25%) cells, and (2). low efficiency

(~10%) but inexpensive to fabricate cells. In the case of high efficiency cells, Olson and

Friedman [1] have tabulated a numb of material systems. These include:

GaAs-GaInP, CuInSe/CdS, and GaAs-CuInSe2 cells. CuInGaSe (CIGS) cells are also

investigated.

Amorphous Si cells are used in portable electronics. Other popular materials include

Si ribbon or polycrystalline Si.

6.1.3 Concentrated Solar Photovoltaics (CSP) and Tandem Multi-junction Cells:

High efficiency solar cells are used in conjunction with solar concentrators. Olson and

Friedman [1] suggest that a 1000 MW power plant using 1000X concentration would require

less than 5000 m2 in cell area.

Tandem Solar Cells: Reduction of losses and improving the conversion efficiency:

Multijunction and tandem cells are used in material system, which are expensive to

fabricate. This way we increase their efficiency. Figure 1 shows a tandem cell that is built on

p-GaAs substrate. It has two n-p junctions and one p+-n+ GaAs tunnel junction. The tunnel

junction is between the TOP and BOTTOM solar cells. The tunnel junction provides the

interface between the two cells. This enables a series connection between two n-p cells having

the same series current.

444

The bottom cell is nGaInP/nGaAs-pGaAs. Here, the nGaAs-pGaAs is the

homojunction, and nGaInP-nGaAs is the isotype heterojunction. N-GaInP layer adjoining

nGaAs layer acts as the wide energy gap window region (Eg = 1.88eV or 1.86eV, the value not

specified). The bottom cell absorbs photons between 1.86 and 1.424eV.

Ref: R.A Metzger,

Manufacturing III-V Solar

cells for space Application,

Page 25, Compound

Semiconductor November /

December 1996.

Fig. 1. Two-junction tandem

cell.

The top cell is nGaInP (Eg not specified)-pGaInP (Eg =1.86eV). I assume that it is a

homojunction cell. However, the nGaInP layer has an adjoining n-AlInP layer (0.25micron)

that serves as the window region. This layer may have slightly higher band gap then the nGaInP

layer. The top cell absorbs photons above 1.86eV. The Ohmic contact is made to an n-GaAs

(doping 6 x1018 cm-3) cap layer. It is easier to form a low-resistance contact to GaAs than

GaInP. The antireflection reflection coating is now deposited all over. The inter-contact pad

separation is 3 m. This separation is comparable to the diffusion length of the EHP. The top

contacts form a grid. An antireflection (AR) coating is deposited over the contact metal grid.

The net open circuit voltage Voc = VocB + VocT, and the current is the same. In a way

this cell has twice the Voc of a single cell. This is schematically shown in Fig .2.

GaAs Substrate Zn-doped

GaInP 0.07um p=3*10^17 cm-3 [Zn]

GaAs 3.5um p=8*10^16 cm-3 [Zn]

GaAs 0.1um n=1*10^18 cm-3 [Se]

GaInP 0.1um n=1*10^18 cm-3 [Se]

GaAs 0.01um n=1*10^19 cm-3 [Se]

GaAs 0.01um p=8*10^19 cm-3 [C]

GaInP 0.6um p=1.5*10^17 cm-3 [Zn]

GaInP 0.1um n=2*10^18 cm-3 [Se]

AlInP 0.025um n=4*10^17 cm-3 [Si]

GaAs 0.5um

n=6*10^18cm-3 [se]

Au

3um AR Coat (2 layer)

Front Grids

Contacting layers

(Eg=1.86ev)

Back Contact

Top Cell

Tunnel

Cell

Bottom Cell

GaInP/GaAs Tandem Cell

445

Fig. 2. I-V Characteristics of a tandem cell.

6.2 Solar Spectrum and Air Mass m

The received power on earth is measured in terms of air mass 'm'. While m is taken to

be '0' for outer space, it has a value of '1' when sun is at zenith. The value of m increases as the

sunlight travels more distance in atmosphere particularly in the morning at dusk, so m is very

high under these conditions. A typical spectrum is shown in Fig .3.

Fig. 3. Solar energy density as a function of wavelength

Air mass is used as a parameter to indicate the variation of solar energy received on earth/outer

space. Definition of air mass:

IL1

IL2

VOC

V

I

VOCB

tandem cell

446

(1)

Table 1. Air Mass m under different conditions

Air Mass Condition Power Density

m=0 Outer space 130 mW/cm2

m=1 Zenith 92 mW/cm2

m=2 is used as more representative 74 mW/cm2

6.3 Absorption of Photons in Semiconductors

Absorption of photons occurs in semiconductors via one of the following electronic transitions:

1. Band-to-band

a. Direct band-to-band with no phonon involvement.

(2)

b. Indirect band-to-band with phonon absorption or emission.

(3)

(phonon absorption) + (phonon emission)

where:

(4)

Ep = phonon energy

α(hv) = the absorption coefficient at photon energy hv.

2. Excitonic transitions

a. Direct transitions

(5)

Here, Eex is the exciton binding energy and

(6)

zenithat is sun the whenlength path

tmeasuremen of timeat thesunlight theof length path opticalm

)E-A(h= 2

1

g

e-1

)E-E-C(h+

1-e

)E+E-C(h=)(h

kT

E-

2

pg

kT

E

2

pg

pp

)h-E(

B|P|+

)h-E(

B|P|

CWnmh6

emmM=C

2

m

v

2

mi

2

o

c

2

io

o2o

72

2he

sinh/2/1 eAEexex

E-h

E=

g

ex2

1

447

(7)

b. Indirect transitions

(8)

For h-Eg + p>>Eex

(9)

3. Absorption in heavily doped semiconductors

a. Band Filling -- Burstein-Moss Shift

(10)

where,

(11)

ζ = Fermi energy

b. Band Tailing

(12)

where:

ζp = fermi level in the valence as measured from the top of the valence band

Ev = an energy in the valence band

The density of states in the conduction band are proportional to 1, (Eo is

an empirical parameter).

(13)

c. Free carrier absorption (intra-band transitions)

(14)

where:

A1λ1.5 = acoustic phonons

h8

em=E 22

r2o

4r

ex

)E+EE-(h 2

1

expgex

)EE-h2

pgex (

)]E(f-[1 |)(h=)(h ceundoped

e+1

1=)E(f

kT

-Ece c

dEe)E( A=)(h E

E

2

1

v

-h

o

p

p

eE

E

o

dxex-

2

1e)EA(=)(h x-

2

1Eo

p

02

1

Eo

h

2

3

o

3.5

12.5

11.5

1free C+B+A=)(h

448

B1λ2.5 = optical phonon scattering

C1λ3.5 = ionized impurity scattering

4. Band to localized impurity

(15)

where:

NI = number of ionized impurities (acceptors)

EA = acceptor energy

Ionized energy for the impurity ground state:

(16)

5. Acceptor to Donor Transitions

It is evaluated in a manner similar to direct band-to-band transition. The factor A is

modified due to the difference in density of state (of the acceptor/donor levels).

Absorption Coefficient Figure 4 shows the plot of α(hv) for various semiconductors. In general, direct gap

semiconductors exhibit α(h) vs h plots which show a rapid increase in absorption once hv>Eg.

In Figure2, Si and Ge are the indirect gap semiconductors. Remaining materials have direct gaps.

Fig. 4. Absorption coefficient of various semiconductors as a function of function of wavelength

and photon energy.(S. M. Sze, Physics of Semiconductors, Wiley-Interscience, New York, 1981)

h

)E+E-(ham2+1

)E+E-(hAN=

2

Ag2*

e

4

2

1

AgII

a8

e=E *

or

2

I

449

Notice that α(hv) increases gradually as hv becomes greater than Eg. In contrast, the

absorption coefficient α(hv) increases rapidly as a function of hv. Inspect the Germanium (Ge)

data. Notice that germanium has an indirect gap at Eg = 0.67 eV. However, it also has a direct

gap at Eg = 0.88 eV and α(h) rises rapidly as h approaches 0.88 eV. Once the absorption

coefficient α is known as a function of hv, we can determine the EHP formation and associated

electrical properties.

Intensity of Light

Once α(hv) is known, we can determine the intensity of light inside a medium. For

example, if the incident photon flux on a surface is Φo, Φ(x) at a distance x is expressed (and

shown in Fig. 5) as

(17)

(18)

This assumes that Φo is transmitted entirely into the medium and there is no surface reflection.

Example 1 (a) Determine the power absorbed in a 10m thick Si sample illuminated by a

10mW photon source. Assume the source to be monochromatic and emitting photons with

energy h = 1.5eV. Given the reflectivity of Si to be 0.31, absorption coefficient (1.5eV) =

800 cm-1 and index of refraction nr = .

e=(x) x-o

eI=I(x) x-o

45.39.11

Fig. 5. Photon flux as a function of x

450

Fig. 6. Photon absorption in Si

a) Find out the portion of the absorbed energy that is not used up in electron-hole

pair (EHP) generation.

b) Determine the thickness of the antireflection coatings for the front and the back

surfaces. Assume the index of the coating material is 1.8.

Solution

a) R1 = R2 = 0.31

Power inside the Si wafer = Pin’(x=0) = Pin - Pin * R1

= Pin (1 - R1)

= Pin (1-0.31)

= 10mW * 0.69 = 6.9mW

Power reading at x=d = Pout’ = Pin’[exp (-d)]

= 6.9mW*exp (-800*10*10-4)

= 3.1mW

Power absorbed = Pin’ – Pout’ = Pabs1 = 6.9 – 3.1 = 3.8 mW.

Optional:

Some of the power that reaches x=d is reflected back and its value is

Pout’’ = Pout’ * R2 = 0.31 * 3.1 mW = 9.61x10-4W

Of this, the fraction absorbed (in tracel to surface R1) = Pabs2

= Pout’’(1-e-d)

= 9.61x10-4 (1-e-0.8)

= 5.29x10-4=0.529mW.

Pout’’ will be reflected from surface #1 (R1Pout’’) and some of this will be

absorbed. But these magnitudes are smaller.

Total power absorbed = Pabs = Pabs1 + Pabs2

Pin = 10mW

(hv=1.5eV)

Front surface Reflectivity

R1 = 0.31

Basic Surface Reflectivity

R2 = 0.31

Pout

10um

Si

0 d

X

451

= 3.8 + 0.529 = 4.329 mW.

For simplicity, this problem will assume that Pabs1 = 3.8mW for simplicity.

b) Portion of photon energy not used up in EHP generation.

Photon energy required to generate an EHP = 1.1 eV in Si

Excess energy per photon = 1.5eV – 1.1eV = 0.4eV.

Number of Photons absorbed / sec =

Excess energy not used / sec = 1.58 * 1016 * 0.4 * 1.6 * 10-19 = 1.01 mW

c)

Fig. 7 Antireflection coating.

For l = 1,

6. 4. Photovoltaic Effect

We would like to explore the operation of a p+-n junction under sunlight. Figure 8 and

Fig. 9 describe the carrier distribution in equilibrium and illuminated conditions, respectively.

sec/10*58.110*6.1*5.1

10*8.3 16

19

3

1 photonsh

Pabs

nr2=1.8

Si

nr=3.45nair=1

AR coating

t

)12(4 2

ln

tr

meVh

8266.05.1

24.124.1

)12(8.1*4

10*8266.0 4

lt

Amt 11481148.08.1*4

10*8266.0 4

452

Concentrations:

p+ region n-region

NA~1020cm-3 = ppo ND=nno=1017cm-3

npo 2.25 pno=2.25x103 cm-3

Fig. 8. Equilibrium carrier concentration in a p+-n junction.

Fig. 9. Minority carrier (hole) distribution in an open-circuited p+-n solar cell.

For simplicity, we are considering the case of an open-circuited cell. In a dark (equilibrium) p+-

n junction, the built-in voltage

453

, (19) (20)

And under forward biasing (VF)

, (21) (22)

Equations (20) and (22) give

(23)

In an open-circuited illuminated p+-n cell we can write using the above analogy:

(24)

(25)

we can express , as seen in Figure5, in terms of

(26)

6.4.1 Qualitative explanation

The concentration gradient (diffusion current) and electric field (drift current) are in

balance in a p-n junction under equilibrium. Each process causes qVbi change in energy of a

carrier, but in opposite direction. The shining of light disturbs this balance. Under open-circuit

condition, no external or internal current can flow. However, the new balance is reflected in the

form of an open-circuit voltage Voc making p+ side positive and n-side negative.

If we connect an external load resistor RL, the current flows in the external circuit and the

terminal voltage reduces from Voc to a different value V depending on the load. The carrier

distribution is shown in Fig. 10. The holes represented by the shaded regions are extracted under

the influence of E-field in the junction. In reality, holes diffuse to the x=Wn (or xn as used in p-n

junctions) boundary. After reaching the boundary they are swept by the E-field. This constitutes

Jp, the hole current density.

p

p

q

kT=V

no

po

bi ln ep=p kT

Vq

nopo

bi

p

p

q

kT=V-V

e

po

Fbi ln ep=p kT

)V-Vq(

epo

Fbi

ep=p kT

Vq

noe

F

epp kT

Vq

no

oc

e

oc

)d(he)(hN+p=)W=p(x=p )W+L(-incphpE=hnon

oc

enp

g

(x)ocp p=)p-p(

oc

no

oc

e

ep=(x) )W-(x-ococp

n

454

Fig. 10. Minority carrier distribution for a given RL

The voltage polarity and current directions are shown in Fig. 11. The exact V-I equation

derivation involves the knowledge of new p(x) distribution which is, in turn, a function of RL.

Fig. 11. Voltage polarity and direction of current flow

6.4.2 V-I Equation: Equivalent Circuit Approach:

Also see Section 6.13 for a rigorous derivation of short circuit current and I-V equation.

Generally, the behavior of a solar cell is modeled by the following equations.

(27)

Here, Is is the reverse saturation current

Isc or IL is the current when the cell is short circuited. That is, V=0 as explained below. In

practice, I turns out to be negative (as we can see from Equation (27)). It signifies that I flows in

the opposite direction than shown in Fig. 12.

)I (orI-1)-e(I=I scLkT

qV

s

Is =qADpPno

Lp+qADnNpo

Ln

455

Fig. 12. Equivalent circuit model of a solar cell.

Short Circuit Current Isc or IL

Generally transitions which are predominant in photon absorption in solar cells are of

band-to-band type. In these transitions the absorption of a photon is accompanied by the

formation of an electron-hole pair (EHP). The collection of photo-generated minority carriers

results in the short circuit current Isc. Thus, Isc is the light generated current. It is also referred as

the short circuit current Isc. It is expressed as

(28)

(29)

Here, Q(hv) = collection efficiency of the generated electron-hole pairs (EHPs), and Nphhv(x) is

the number of photons having an energy hv at a point x.

Equation (28) assumes that each photon absorbed results in the formation of an electron hole

pair. The optical generation rate (gop = number of photons incident per unit area per second) is

expressed as

(30)

The excess carrier concentrations are:

(31)

(32)

In an n-type material ND=no, ; (with no >> po). The excess minority carrier

concentration is δp.

(33)

))d(h(hN)Q(hq=I phEscg

e(0)N=(x)N)x(h-h

phhph

n)p+n(=g oorop

)p+n(

1=

oor

n

noppn g==

N

n=p

D

2i

o

n

1=

)p+n(

1=

oroor

p

456

The number of holes created in n-region within Lp (on per second basis) is ALpgop.

Assuming that gop is constant, the short circuit current is:

(34)

Where:

A = area of cross-section.

IL = Total light generated current due to all the carriers generated by photon

absorption, and it is greater than Isc.

6.4.3 Open Circuit Voltage Voc, Maximum Power Point (Vm, Im) and Fill Factor (FF)

Substitute I=0 in Eq. (27)

(35)

and V=Voc, so

(36)

Generally, IL>> Is (Is being the reverse saturation current), and Voc can be simplified as

(37)

Maximum Power Transfer Point (Vm, Im)

Power delivered to the load RL=VI

(38)

The power is expressed as

)L+L(qAg=I=I npopscop

ppp D=L

nnn D=L

)I (orI-1)-e(I=I scLkT

qV

s

I-1]-e[I=0 LkT

qV

s SCLkT

qV

II-eIs )1( I = sI (qV

kTe -1)- IL

SCkT

qV

IIs-eIs ¶I

¶V= Is

qV

kTe .q

kT

SCkT

qV

IIseIs

I

I+I

q

kT=V

s

sLoc ln

I

I

q

kTV

s

Loc ln_

Is

IIse

SC

kT

qVoc )ln()ln(

Is

IIse

SC

kT

qVoc

)ln(Is

IIs

kT

qV SCoc

VI=P

457

(39)

Equation (38) can be rewritten as

, (40a), or (40b)

Graphical Solution: Determination of Vm, Im

Let Q be any operating point on the solar cell characteristic. is the slope of the straight

line drawn between (V,0) and (0,-I) points. is slope of the tangent drawn at (V,I). When the

two slopes are equal, i.e. two lines are parallel, we satisfy Equations (40a) and (40b). The

maximum power point is at which the values of the two slopes are equal (see Fig. 13).

Fig. 13. Determination of maximum power point from a solar cell characteristic.

Mathematical expressions for Vm and Im : Maximum power transfer point

Using Equations (27) and (40b)

(41)

Note that V=Vm in Equation (41)

(42)

II

V+V=0=

I

P

I

V-=

I

V

V

I-=

V

I

V

I-

V

I

V

]I-1)-e(I[-=e

kT

qI

LkT

qV

skT

qV

s

V

]I-I-eI[-=e

kT

qI

m

LskT

Vq

skT

Vqs

m

m

458

Equation (36) can be used to eliminate IL from Equation (42)

(43)

Dividing Equation (42) by Is, and multiplying by Vm.

(44)

Equations (43) and (44) simplify to

(45)

(46)

Taking natural logarithm

(47)

(48)

Equation (48) can be solved numerically. Substituting Vm in Equation (27) results in an

expression for Im

(49)

Fill Factor: FF

The fill factor is related to the shape of the V-I plot. It determines the overall maximum

power output for a given load. It is defined as the ratio of inner rectangle (related to maximum

power output) and the outer rectangle (maximum possible power output) as shown in Fig. 14.

(50)

e=I

I+IkT

Vq

s

Ls oc

I

I+I+e-=e

kT

Vq

s

LskT

Vq

kT

Vqm mm

e+e-=ekT

VqkT

Vq

kT

Vq

kT

Vqm ocmm

e=kT

Vq+1e kT

Vqm

kT

Vq ocm

kT

Vq=

kT

Vq+1+

kT

Vq ocmm

ln

kT

Vq+1

q

kT-V=V

mocm ln

I-1]-e[I=I LkT

Vq

sm

m

rectangle outer of Area

rectangle inner of Area=

IV

P=

IV

IV=FF

Loc

m

Loc

mm

459

Fig. 14. I-V Fill Factor curve.

6.5 Conversion Efficiency and Losses

The conversion efficiency is defined as the ratio of maximum electrical power output to

the optical power incident. It is defined as

(51)

To optimize the conversion efficiency ηc, we need to maximize FF, Voc and IL. Note that

Voc increases with Eg of the solar cell material. On the other hand, IL increases as Eg is decreased.

6.5.1 Series and Shunt resistances in the equivalent circuit

The equivalent circuit and the current-voltage equation characterizing the solar cell get

modified in order to account for the series and shunt resistances.

Series resistance RS is contributed to by:

a. Bulk resistance of n+ and p regions (in a n+ -p cell).

b. Ohmic contact resistance (Ohmic contacts are needed in order to attach leads; in Si

aluminum is used to form Ohmic contact to p-regions and arsenic-doped gold is used

for n-region; many times a diffusion of n+ type is done prior to ohmic contact

evaporation in n-type materials).

Shunt resistance RSH represents leakage across the junction. The leakage current is caused by

defects, traps, and surface states (in an un-passivated device). These are shown in Fig. 15.

P

IV=

Power SolarIncident

IV=

in

mmmm

c

P

IVFF=

P

IV

IV

IV=

in

Loc

in

Loc

Loc

mm

460

Fig. 15. Equivalent circuit with series RS and shunt RSH resistances.

Node A: Kirchhoff Current Law (KCL)

The I-V equation is modified to:

(52)

The series resistance, RS, shows up in the I-V Characteristics of a solar cell, shown in Fig. 16.

Fig. 16. Influence of RS on I-V behavior.

V-I Equation including series and shunt resistances

The characteristics of p-n junctions including the effect of recombination current (in the junction)

include:

L

SH

SkT

IRVq

S IR

IRVeII

s

)1(

)(

RS2

RS1

IL1

IL2

VOC

V

I

I L

I D

R S h

R S I

R L V

0 D

SH

S L

R

IR V I I I

461

(53)

(54)

(55)

An ideality factor is included in Equation 53 to make Equation 53 match the experiment.

(56)

Losses : Losses in a solar cell are due to: a. Long wavelength (h < Eg of 1.1eV for Si)

b. Excess photon energy not used in generating electron-hole pairs.

c. Voltage factor (qVOC/Eg)

d. Fill Factor

Mechanisms responsible for various losses are listed in Solar Design Section. These will be

discusses in the solution set. Additional losses include collection efficiency, series resistance

loss and shunt resistance loss.

6.6 Solar Cell Materials and Technologies: Generation I Flat Plate Arrays

Silicon Based

1. Single crystal wafers

2. Ribbon

3. Polycrystalline cast ingots

Thin Films

1. Cu2S/CdS polycrystalline Films

2. GaAs thin layers on Sapphire ribbons

3. Amorphous Si and chalcogenide glasses films on steel

4. Doped polyacetylene and other semi conducting polymers

Arrays with Solar Concentrators

1. Silicon wafers (30-50X)

2. Gallium arsenide (50-200X)

3. Graded heterostructure (50-5000X): thermal photovoltaic

4. Tandem cells Multi Junction cells

Semiconductor Materials

Morphology Form

Single Crystal Ingot/Wafer .005" - .015"

Ribbon 0.010"

Thin Films 0.0001 - 0.001"

Polycrystalline Same as single crystal

Amorphous Thin films grown on cold substrates (film

thickness 0.0001")

1)-e(I+1)-e(I=I 2kT

qV

SRkT

qV

Diffs,

r

iSR

2

nqAWI

]L

ND+

L

PDqA[=I

n

pon

p

nopDiffs,

1)-e(I=I kTn

qV

Diffs, 1

462

Conversion efficiency as a function of energy gap:

Conversion efficiency is related to the energy gap of the semiconductor used. This is

shown in Fig. 17. Also see question #6 in the problem set at the end of this Chapter.

Fig. 17. Efficiency vs. semiconductor energy gap.

In the case of amorphous materials, p-I-n and Schottky interfaces are used to fabricate

solar cells. Typical structures are shown in Fig. 18.

Fig. 18. Cross-sectional schematic of p-i-n junction and Schottky barrier solar cells using a-Si.

g

sc

goc

EI

EV

1

463

The energy band diagram of a heterojunction solar cell, bar chart of various losses, and the

absorption coefficient for amorphous Si are shown below in Fig. 19. Fig. 19(a) shows the energy

band diagram of a GaAs homojunction cell with a AlGaAs window to reduce losses in the

window region. Fig. 19(b) shows various losses in Si, GaAs and heterojunction cells.

[Reference. Huber and Bogus, 10th IEEE Photovoltaic Specialists Conference, p. 102, 1973]. Fig.

19(c) compares the absorption coefficient in amorphous and crystalline Si.

Fig. 19. (a) Heterojunction cell, (b) losses in different devices, (c) absorption in c-Si and a- Si.

464

Solar Cell Structures Processing Steps for Single Crystal Solar Cells (Silicon)

Figure 18 shows a typical cycle to process Si ribbons from molten Si. The ribbons are

no longer used.

Fig. 20. Processing cycle for Si cells using wafers and ribbons.

Requirements of Si per KW:

Silicon powder 75LB Crystal Ingot 15LB Cut and Polished Wafers 7LB

Silicon powder 15LB Single Crystal Ribbon 8LB

Amorphous Thin Films on Steel or Glass Substrates About 16/100 Lb (03 oz)

Cost of Pure Silicon Powder

$27 per pound (70’s numbers). Potential $2.20 per pound based on Stanford Research

Institute Process

465

6.7 Solved Examples

Example 2

The following figure shows an n+-P Si diode with following device / material parameters.

Fig. 21. An abrupt n+-p Si solar cell (the light is from the left or n+ side)

Given:

n+-side:

Donor concentration ND = 1020 cm-3, minority hole lifetime p=2x10-6 sec.

Minority hole diffusion coefficient Dp=12.5cm2/sec.

p-side:

Acceptor concentration NA=1016 cm-3, n=10-5 sec. Dn=40cm2/sec. Junction Area =

A = 1 cm-2, ni (at 300K)=1.5x1010cm-3, r (Si)= 11.8, 0=8.85x10-14F/cm, =r0.

Assume all donors and acceptors to be ionized at T=300K.

c) Determine the open circuit voltage at AM1 (this produces a short circuit current

of IL=Isc=27mA).

c) Determine the maximum output power.

c) Find the fill factor FF.

Solution

a)

Reverse saturation current =

n+ P

I

-Xn Xp

0

X

s

scsoc

I

II

q

kTV ln

p

np

n

pn

sL

pqAD

L

nqADI

00

466

np0 =

pn0 =

Ln =

Lp =

Is = 7.2pA.

Open circuit voltage at 300K,

c) Pm = VmIm

Fig. 22. Im-Vm point shown on the solar characteristics (curve is not joined smoothly).

(A)

Find Vm: Write a program or do by trial & error. Assume as a trial Vm = 0.5

1. 0.5 (LHS) => 0.57-0.07798 (RHS)

34

16

202

10*25.210

10*25.2 cmN

n

A

i

32

25.2 cmN

n

D

i

cmD nn

25 10*210*40

cmD pp

36 10*510*2*5.12

voltsVoc 57.05709.010*2.7

10*2710*2.7ln0259.0

12

312

0.5

-10

-20

-30

VOC

= 0.57

(Im

,Vm

)ISC

=27mA

0.40.30.20.1

kT

qV

q

kTVV m

ocm 1ln

kT

qVV m

m 1ln0259.057.0

467

=>0.492

2. Next guess Vm = 0.49

0.49 (LHS) => (RHS)

3. Vm is slightly higher than 0.49V.

Vm = 0.492 makes both LHS & RHS almost the same.

Get Im by substituting in Vm in I-V equation.

c) Fill Factor (FF) =

Example 3: Plot V-I characteristics for the cell of Fig. 21 using the analytical equation I = Is

[exp (qV/nkT)-1]-Isc using n=1 and n=1.5. Which one of these two solar cells has a higher fill

factor. (The one having n=1 or with n=1.5)

Solution: Plot the following 2 equations.

For n=1, (B)

For n=1.5, (C)

V4925.00259.0

49.01ln0259.057.0

SCkT

qV

sm IeIIm

1 mAe 72.2510*27110*2.7 30259.0

492.0

12

mWmAPm 65.12492.0*72.25

822.027*57.0

72.25*492.0

mA

mA

IV

IV

SCOC

mm

312 10*27110*2.7

kT

qVm

eI

35.112 10*27110*2.7

kT

qVm

eI

For n=1 (Equation B) For n=1.5 (Equation B)

I = 27mA V = 0

I = 0 V =VOC = 0.57V

I 27mA V = 0.2V

I = 26mA V = 0.3V

I = 26.16mA V = 0.4V

I = 25.25mA V = 0.5V

I = 23.2mA V = 0.52V

I = Im = 25.72mA Vm = 0.492V

I = 18.8mA V = 0.54V

I = 27mA V = 0

I = 0 V =VOC = 0.856V

I 27mA V = 0.2V

I = 26.96mA V = 0.6V

I = Im = 25.75mA Vm = 0.74V

I 27mA V = 0.54V

I = 26.55mA V = 0.7V

I = 24.9mA V = 0.76V

I = 21.17mA V = 0.8V

I = 10.7mA V = 0.84V

Fill Factor (FF) =

Fill Factor (FF) =

822.0

57.0*10*27

492.0*10*72.253

3

823.0856.0*10*27

74.0*10*75.253

3

468

Fig. 23. V-I characteristics for different n values.

Example 4: For the cell of Fig.21 determine the Voc, Pm and fill factor when the solar light is

concentrated by 100 times over the AM1 value.

0

Solar concentration (AM1 Suns)

1 10 100 10000.5

0.6

0.7

0.8

14

16

18

20

22

24

0.70

0.75

0.80

105

103

EF

FIC

IEN

CY

ŋ (

%)

VO

C (

V)

FIL

L F

AC

TO

R

JS

C(m

A/c

m2)FILL FACTOR

JSC

PROJECTED ŋ

MEASURED ŋ

VOC

101

Fig. 24. Jsc and other parameters as a function of concentration at AM1.

0.5

-10

-20

-30

VOC

= 0.57

(Im

= 23.2mA,

Vm

= 0.52V)

ISC

=27mA

0.40.30.20.1

n = 1 n = 1.5

FF = 0.822 FF = 0.823

(Im

= 25.75mA,

Vm

= 0.74V)

VOC

= 0.856

469

HINT: Find the short circuit current density Jsc using the plot of Fig. 24.]

JSC at AM1 * 100 from the plot = 3*103 mA/cm2.

The short circuit current ISC = IL = 1 cm2 * 3 * 103 mA/cm2 = 3*103 mA = 3Amp.

Pm = ImVm

Select Vm (below 0.693V) such that LHS equals the RHS.

Guess values:

1. Vm = 0.59V

0.59 (LHS) => (RHS)

2. Vm = 0.61V

0.61 (LHS) => (RHS)

Vm = 0.61V

Fill Factor (FF) =

Fill factor improved slightly.

6.8 Solar Cell Design Two design examples are presented one each for Air Mass m=1 and air mass m=0.

6.8.1 Solar Cell Design for p+-n Si solar cell for terrestrial applications Air Mass m=1.

Given: Material = n-Si, Resistivity = 10Ω-cm (see tables to find ND), average power

incident Pin for air mass m = 1 is 92.5 mW/cm2 (See Fig.25, Ref. Sze, Page 289)

V693.010*2.7

10*10*3ln0259.0

I

Iln

q

kT V

12

33

S

SC

100*1AMOC

kT

qV

q

kTVV m

ocm 1ln

V61.00259.0

59.01ln0259.0693.0

V61.00259.0

61.01ln0259.0693.0

SCkT

qV

sm IeIIm

1

A878.231e10*2.7 0259.0

61.0

12

W756.161.0*878.2Pm

844.03*693.0

756.1

IV

IV

SCOC

mm

470

Specification desired: Fill factor ≥ 0.9 and power conversion efficiency ≥ 12%.

1) Evaluate the surface reflection loss. How would you design an antireflection

coating to eliminate this loss (provide index and thickness of this coating).

2) Determine the optimum loading condition (Vmp, Imp), and compute fill factor for

your cell and show that it is > 0.9 (change the ideality factor n if the fill factor is <

0.9).

3) Evaluate the other important losses and show that cell would be over 12%

efficient.

4) Determine the doping concentrations and minimum thickness of n-region.

Assume: Diffusion lengths (Ln, Lp) and minority lifetimes (τn, τp) as given in previous

problems.

Given that the p+ region is 0.25 micron thick and having a doping level NA = 1020

cm-3. IL or ISC: The photo generated current IL (or ISC) decreases as a function of

the semiconductor band gap Eg. The open circuit voltage increases with Eg.

JSC = JSC0-a(Eg-E0)2 (We have a better equation, see Question #6).

JSC0 = 50 mA/cm2

E0 = 0.5eV

a = 64 mA/(eV)2 or use the Jsc vs Eg plot in Fig. P8.

Fig. 25. Solar spectrum at AM0 and AM1.

471

Fig. 26(a). Solar cell characteristics under dark and illuminated conditions. Fig. 26(b)

shows the short circuit current Isc as a function of energy band gap Eg.

(i)

Isc

VOC

V

I

(a) The current voltage curve of a p-n junction solar cell

(i) in the dark and (ii) under illumination

Imp

(ii)

Vmp

0 0.5 1.0 1.5 2.0 2.53.

0

10

(b) The maximum possible short circuit current Imax for solar cell

10

0

Eg (eV)

Fig. 26(a). Solar cell characteristics under dark and illuminated conditions. Fig. 26(b). Isc as a function of Eg.

472

Design Solution

Fig. 27 A Si p+ - n solar cell.

1) AM1 => Pin = 92.5 mW/cm2

Reflectivity of Si = R =

Reflection Loss = Pin * 0.3 = 27.75 mW/cm2.

Antireflection Coating (AR)

Fig. 28. Schematic of the solar cell.

λav = 0.64μm or hυav = 1.9375eV

NA=1020cm-3

ND=4x1014cm-3

nr

cm

n-Si

Pin

0.5m

435.38.11 Sir,, Sirn

3.01435.3

1435.322

airr

airr

nn

nn

nr=3.435

cm

n-Si

0.5m

Si

AR Coating

Junction

473

Index of AR coating = nr2 =

tAR = 863.5 Å

2) Vmp, Imp

Vmp = Vm =

ISC at air mass 1 for Si.

IS = Reverse saturation current =

For n-side,

For p+ side,

Vm = Vmp

853.1435.3* rair nn

1l ,124

2 ltn avr

mm

nt

r

0863.0853.1*4

64.01

4

m0.64

2

kT

qV

q

kTV m

OC 1ln

S

SSCOC

I

II

q

kTV ln

mAmAI AMSC 97.383.135

5.92*571,

n

pon

p

nop

L

nD

L

pDqA

35

14

2102

10*625.510*4

10*5.1 cmN

np

D

ino

cmDL ppp

36 10*510*2*5.12

3

20

2102

25.210

10*5.1 cmN

nn

A

ipo

cmDL nnn

35 10*410*40

AI S

10

3

53

10*25.210*5

10*625.5*5.12*1*10*6.1

VI

I

q

kTV

S

SCOC 491.0

10*25.2

10*97.38ln*0259.0ln

10

3

474

Solution using trial and error (making a guess and getting left hand side = right

hand side). Let Vm = 0.45V

LHS RHS

0.45 0.415

0.43 0.416

0.42 0.417

0.418 0.4174

0.4175 0.4174

Vmp = Vm = 0.4175V

Fig .29. V-I characteristic.

0259.01ln0259.0491.01ln0259.0491.0 mm

m

V

kT

qVV

SCkT

qV

smp IeII

1 mAmAe 72.3697.38110*25.2 0259.0

4175.0

10

mWPm 33.154175.0*10*72.36 3

0.2 0.4 0.6

-10

-20

-30

-40

VOC

=0.491V

Im

=36.72mA

Vm

=0.4175V

ISC

=38.97mA

475

Fill Factor (FF) =

Change of ideality factor n

From the previous examples, we can say that ‘n’ does not change fill factor.

There are some ways to obtain a fill factor of 0.9.

a. One-way is to reduce Is by increasing the doping of the substrate.

b. The other way is to use a heterojunctions. Both results in high VOC.

Reducing Is or Increasing VOC

Increase the doping of the substrate. Let ND = 100*4*1014 = 4*1016 cm-3. We will

keep NA for p+ = 1020 cm-3.

VOC value will go higher if ND is increased above 4*1016.

The new Is’ value

The new equation is,

Guess values are,

LHS RHS

0.55 0.5302

0.53 0.531

0.531 0.5311

8.097.38*491.0

33.15

mA

mW

SCnkT

qV

smp IeII

1

VI

II

q

kTV

S

SSCOC 6106.0

10*25.2

10*97.38ln0259.0ln

12

3

p

np

L

pqAD '

0

33'

0 10*625.5 cmpn

AI s

12

3

319' 10*25.2

10*5

10*625.5*5.12*10*6.1

SCkT

qV

IeI

110*25.2 12

0259.01ln0259.06106.01ln

''

'' mmOCmmp

V

kT

qV

q

kTVVV

VVm 531.0'

mAIm 16.3710*97.3810*8033.1 33'

476

Fill Factor =

Fig. 30. I-V plot for new ND = 4*1016 cm-3 shows a slight improvement.

3) Losses: Losses in a solar cell are due to:

e. Long wavelength (h < Eg of 1.1eV for Si)

f. Excess photon energy not used in generating electron-hole pairs.

g. Voltage factor (qVOC/Eg)

h. Fill Factor

Long wavelength photons are not absorbed as their energy is below the

energy gap Eg.

a. With reference to the figure 31, in area of region 1, 2 & 3, the solar power is:

b.

c. Region #1 68.4 W/m2

d. Region #2 72.69 W/m2

e. Region #3 35.77 W/m2

f.

g. Total long wavelength (h < Eg of 1.1eV for Si) photons losses= 176.86 W/m2

83.010*97.38*6106.0

10*16.37*531.03

3

0.1 0.5

-10

-20

-30

-40

VOC

=0.6106V

Im

=37.16mA

Vm

=0.531V

ISC

=38.97mA

0.3

477

Fig. 31. Two curves related to solar spectral irradiance. Fig. 32. Excess energy loss(below).

478

h. Excess photon energy for Air Mass One

Excess photon energy = h - Eg,Si

Let AM1 plot above < 1.1m or h > 1.1 eV is divided in several regions. (The

exact way is to find the area under the curve numerically)

AM1 plot above h > 1.1eV

Regions are Triangle J, Trapezoid J, Trapezoid marked as #4, small

rectangular regions #5 & #6.

J => Photon energy at =

Photon energy at J =

Excess photon energy = 3.21 – 1.1 = 2.11eV.

Area of the J =

Excess energy not used =

Trapezoid J => h at J =>

h at =>

Area = Rectangle ’J + ’

= 950*(0.84-0.51) + (0.84-0.51) * (1550-950)/2 = 412.5 W/m2

Excess energy lost = (412.5/1.953) * 0.843 = 178.05 W/m2

Trapezoid #4, Rectangle #5 & Rectangle #6 can be combined by a rectangle.

= -500 * (0.84-1.1) = 130 W/m2

Excess energy = have – Eg = 1.3 – 1.1 =0.19eV

eV431.0

24.1

eV43.251.0

24.1

eVh ave 21.32

43.24

2/1552

31.051.0*1550

2* mW

JJ

2/10111.2*21.3

155mW

eV43.271.0

24.1

eV476.184.0

24.1

eVh ave 953.12

43.2476.1

eVh ave 3.12

127.1476.1

2

1.1

24.1

84.0

24.1

479

Excess energy not used =

Add all excess energy lost without being used as EHP = 101 + 178.05 + 19

298.05 W/m2

c. Voltage factor =

d. Fill factor = 0.8

These four losses are plotted as follows.

Fig. 33. Various Losses

The cell will be above 12

(iv) Doping concentrations & minimum thickness of n-region.

Pabs in n-Si = 925 – 176.86 = 748.14 W/m2

Pabs = Pin (1-e-(have

)d)

Where, d – thickness of n-region.

hav = 1.9375eV, (at 1.9375eV) = 4*103 cm-1

In reality, d 100 – 150 m to absorb h Eg = 1.1eV.

2/1919.0*3.1

130mW

eV

37.9or 379.01.1

4175.0

g

OC

E

qV

100%

81%

48.65%

30.22%

24.17%Fill Factor = 0.8

Voltage Factor = 0.37

176.86

298.05

450.09 W/m2

748.14 W/m2

925 W/m2

Excess Energy(hav

-Eg) loss

Long Wavelength(hav

<Eg)

de *10*4 3

1925

748

de 40001808.0

md 14.4)2.0ln(4000

1

480

6.8.2 Solar cell design for AM 0 (outer space)

Isc Voc as a function of Eg: See also Section 6.12.

Polynomial exponential set for Isc plot

Isc = 100-36.73(Eg-0.3019)-9.779(Eg-.03019)2+3.911(Eg-0.0319)2 (57)

Voc =kT(ln (Isc+Is)/Is)≅ (kT/q)ln (Isc/Is) (58)

=(kT/q)ln (Isc)- (kT/q)ln (Is) (59)

For a p+-n cell

Is=qADpPno/Lp+ qADnnpo/Ln

qADnnpo/Ln is small as npo<< pn0

≅ qADnnpo/LpNd(4)

≅ qADp/LpNd*4*(4∏2mnmp(kT)2)/4Ψ)3/2e-(Eg/kT)

=((1.6*10-19*10-4*12.5*10-4)/(5*10-3*10-2*10-16*10-6))* ((4∏2mnmp(kT)2)/4Ψ)3/2e-

(Eg/kT))

= 2.12*105*2e-(Eg/kT) (60)

Eqs. (60) and (57)

Voc = kT/q ln Isc-kT/q*ln(2.12*105*e-Eg/kT)

=kT/q ln Isc-kT/q ln2.12*105+Eg/q

The attached plots that Isc,Voc, Isc Voc we can see Isc Voc peaks at~ 1.4eV

ni2 = 4*(((4∏2*.0067*.62*(9.11*10-31)2*(1.38*10-23*300)2)/(6.639*10-34)4)3/2

Isc = 100 – 36.73(Eg – 0.03019) – 9.779(Eg – 0.03019)2 +3.911(Eg – 0.03019)3

Fig. 34 shows Isc as a function of Eg at Air mass m=0. See also Fig .17(b) for ISC data.

481

Fig. 34. Isc as a function of Eg (Air mass 0).

Fig. 35. Voc as a function of Eg (Air mass 0).

Voc = 0.0259ln(Isc) – 0.317 + Eg/q

482

Fig. 36. Voc x Isc as a function of Eg (Air mass 0).

Solar losses for Air Mass AM0

Fig. 37. Solar spectral irradiance air mass 0

0.2 2.62.01.40.8

200

0

2200

2000

1800

1600

1400

1200

1000

800

600

400

1.1 1.7 2.3

Wavelength (m)

Sp

ectr

al

Irra

dia

nce (

W/m

-2

m-1

)

2400

Air Mass Zero (353 W/m2)

GaAs (c = 0.87 m)

Si (c = 1.1 m)

h1.11.3

A

B

C

J

H

I

E

F

DG

1.4

483

We have divided the ABED part of the spectrum in which photons have energies greater

than 1.1eV (Si = Eg)

Let us look at DEFG trapezoid

No of photons whose power is represented by FE curve.

Excess energy / Photon =

Total excess energy lost per sec =

Similarly find the total excess energy lost per second in other trapezoids. Add then up and

you will obtain the power lost. Find the percentage power lost.

G D

E

F

average

DEFGPH

h

AreaofFEGDN

/

Si ofEh gaverage

hQ h

gaverageDEFGPH EhN *

484

6.8.2.1 Bar Chart of Losses for AM0, m = 0

Fig. 38. Losses on Bar Chart for AM0 Si Solar Cell Series and Shunt Losses are not shown.

Calculations:

The AM0 curve is divided into 5 trapezoids. We need to calculate their area:

E F (or a) D (or b) G = EFDG

GD = 0.95 m 1.1 m (x-axis)

The photon energy is = 1.24/0.95m = 1.3 eV at Point G or Line EG

The photon energy is = 1.24/1.1m = 1.1 eV at Line FD or ab

Average photon energy in this trapezoid = (1.3 + 1.1)/2 = 1.2 eV.

Excess energy per photon in DEFG = 1.2 – 1.1 = 0.1 eV.

Number of photons in the trapezoid DEFG = Area/1.2 eV = Shaded Area/1.2 eV.

EFDG Shaded Area = Rectangle + Triangle

= 600(1.1 – 0.95) + (EE’ × E’F)/2

= 90 + 0.15(850 – 600)/2 = 90 + 19.75

= 108.75 W/m2

Excess Energy = 0.1(108.75/1.2) = 9.06 W/m2

485

HEGI Trapezoid II

HI line: value = 0.71 m; h = 1.746 eV

EG line: value = 0.95 m; h eV

Average (h photon energy = (1.746 + 1.3)/2 = 1.523 eV

Excess energy = 1.523 – 1.1 = 0.423 eV

Area of HEGI trapezoid = Rectangle H’EGI + Triangle HH’E

= 850(0.95 – 0.71) + [(1300 – 850)(0.95 – 0.71)]/2

= 204 + 54 = 258 W/m2

Excess Energy = 258(1.523 – 1.1)/1.523 = 71.65 W/m2

BJIH Trapezoid III

BJ line: value 0.51 m; h = 2.43 eV

HI line: value = 0.71 m; h eV

Average photon energy = (1.746 + 2.43)/2 = 2.088 eV

Excess energy = 2.088 – 1.1 = 0.988 eV

Area of BJIH trapezoid = Rectangle B’HIJ + Triangle BB’H

= 1300(0.71 – 0.51) + 0.2(2100 – 1300)/2

= 260 + 80 = 340 W/m2

Excess energy = 340(0.988)/2.088 = 160.88 W/m2

Triangle ABJ

Area of ABJ triangle: 2100(0.51 – 0.2)/2 = 325.5 W/m2

Average photon energy = 1.24/0.2 – 1.24/0.51 = 6.2 – 2.43 = 3.77 eV

Excess energy = 3.77 – 1.1 = 2.67 eV

Excess photon energy wasted = (325.5)(2.7)/3.77 = 230 W/m2

Total excess energy wasted = 9.06 + 71.65 + 160.88 +230 = 471.59 W/m2

486

6.9 Fabrication & Simulation of Solar Cells: Student Project Johnhenri Richardson

Advisor: Professor Facquir Jain Research Partner: Pratiba Anand

UConn Dept. of Electrical & Computer Engineering REU Summer 2006, August 2, 2006

Abstract

A single junction silicon solar cell was fabricated by phosphorus diffusion of a p-type

silicon wafer. Additionally, two-junction solar cells were modeled in MATLAB. The

fabricated solar cell had a Voc of 0.433V, a Jsc of 9.8mA/cm^2, and a fill factor of 67%. The

simulated tandem solar cell had a Voc of 1.19V, a Jsc of 18mA/cm^2, and a fill factor of 88%.

Introduction

A simple solar cell can be made by fabricating a diode whose bandage energy is near

typical photon energies in sunlight. At the earth’s surface, sunlight intensity as a function of

wavelength peaks around 650nm, or 1.9eV. This makes a silicon p-n junction, with a bandage

energy Eg of 1.1eV, a good material for solar cell fabrication. The silicon solar cell absorbs

photons with energies at or above 1.1eV, creating mobile charge carriers in the form of

electrons and holes. If connected to a circuit, the solar cell can then behave like a current source

out to some open circuit voltage limited by its material properties.

Such a silicon solar cell was fabricated in the lab. Starting with a p-type silicon wafer,

we created a p-n junction by phosphorus diffusion. The I-V characteristics of the resulting solar

cell were promising and could probably be easily improved. However, due to equipment

failure, only one solar cell could be fabricated during the program’s time frame.

Single junction solar cells have two major inefficiencies. First, all photons below the

solar cell band gap energy pass through unabsorbed. These photons do not have enough energy

to excite an electron into the conduction band, so they cannot be used to generate charge

carriers. Second, photons above the band gap energy do not impart all of their energy to the

circuit. Instead, the difference Ephoton - Eg is lost via thermal and other interactions, and only

the bandage energy is used by the solar cell.

To minimize these losses, many researchers turn to tandem solar cells. Tandem solar

cells consist of two p-n junctions stacked and connected in series. The top junction has a

relatively large band gap energy, capturing the high energy photons with minimal thermal

losses while letting low energy photons pass. The bottom junction has a lower band gap energy,

absorbing a large fraction of what the top junction lets through.

These tandem cells were modeled based on the work of Kurtz.1 Using MATLAB, the

tandem solar cell’s I-V characteristics were calculating from material properties of the solar

cell itself. The resulting data allow exploration of tandem cells and cells made from materials

other than silicon.

Solar Cell Fabrication Procedure

One solar cell was fabricated in the lab. We started with a boron-doped p-type silicon

wafer. The wafer was cleaned by a standard procedure to remove oxides from its surface. Once

prepared, the sample underwent phosphorus diffusion at 1000°C for 10 minutes. This deposited

a layer of phosphorus-doped n-type silicon on both sides of the wafer. Back-etching followed

487

diffusion. Protecting the front surface of the wafer with wax, we dipped the wafer into a slow

silicon etch solution, which removed the n-type layer from the back surface. We etched until a

thermocouple probe showed the back surface to be p-type. These steps created the needed p-n

junction.

Following diffusion, the sample underwent metallization. Thermal evaporation was

used to apply metal contacts. The back surface was covered with a layer of aluminum. The

front surface was covered with a pattern of gold-arsenic dots, each 0.51mm in diameter and

spaced 1.3mm apart. This small, tightly spaced dot pattern created complications in then next

step.

Once metal contacts had been applied, the gold-arsenic dots needed to be mesa etched.

The procedure called for each dot to be covered with wax, then a slow silicon etch to be applied

to the top surface. This would leave only a ring of n-type silicon around each dot, minimizing

leakage currents flowing between contacts. However, because the dots were so closely spaced,

applying wax to individual dots proved extremely difficult. When the mesa etching procedure

was complete, many gold-arsenic contacts had either been lost or were still connected to other

contacts by n-type silicon. This made measured results from the solar cell very inconsistent

from dot to dot. Nevertheless, some contacts were properly etched and gave good results.

Figure 38A shows a magnified image of the solar cell’s surface.

Fig. 38A. 20x magnification of the fabricated

solar cell’s top surface. Each gold-arsenic

contact has a diameter of 0.51mm. The two

contacts in the center are properly etched,

while a large n-type silicon mesa connects

three contacts on the right.

Fig. 38B. I-V characteristics of one contact on the

solar cell. Current-axis intercept is the short-circuit

current. Voltage-axis intercept is the open-circuit

voltage.

Unfortunately, the thermal evaporation unit subsequently broke. Repairs could not be

completed within the REU time frame. This prohibited us from making more solar cells during

the program.

Measured Results

Properly etched areas of the solar cell had a Voc of 0.433V and a Jsc of 9.8mA/cm^2.

The fill factor, equal to Pmax / (Voc * Isc), was 67%. These results are consistent with other solar

cells fabricated by the research group and comparable but slightly less powerful than some

similar, commercially available silicon solar cells which have a Voc of 5.5V.2,3 Figure 38B

shows the current-voltage characteristics of a properly etched contact.

488

6.10 Tandem Solar Cells 6.10.1 Tandem Solar Cell Simulation

In addition to fabricating a silicon solar cell, we modeled tandem solar cells in

MATLAB. Using simulation work done by Kurtz at the National Renewable Energy

Laboratory as a guide,1 we wrote a program that took in material parameters of the top and

bottom cell and intensity of incident light and outputted Jsc, Voc, and the fill factor. The program

relied on the following equations, all obtained from Kurtz’s paper.1[S.R. Kurtz, P. Faine, and

J.M. Olson, J. Appl. Phys. 68, 1890 (1990)].

The reverse bias current can be obtained from an array of material parameters: The reverse

saturation current density Jo is

(61)

where e is the charge of an electron, ni is the intrinsic carrier concentration, NA and ND are the

acceptor and donor concentrations, is the minority carrier lifetime, S is the surface

recombination velocity, and x is the layer thickness. D, the diffusion constants, can be

calculated from:

and

where k is Boltzmann’s Constant, T is the temperature, and is the charge carrier mobility.

The photocurrents from the top and bottom junctions can be obtained from two

related equations (62) and (63):

(62) top (63) bottom equation

where I0() is the incident light intensity, is the absorption coefficient, and t is the

thickness of the top cell. Jsc for the whole solar cell is the smaller of the two currents.

Once the currents have been calculated, all other results come from the voltage

equation:

(64)

Voc can be calculated by plugging in J = 0. Output intensity is simply the product of J

and V. The current and voltage yielding maximum intensity, Jmp and Vmp, can be obtained

from the relationships:

hhnhhnhhh

hhnhhnhhh

h

h

D

i

eepeepeee

eepeepeee

e

e

A

i

DxDxDS

DxDxDSD

N

ne

DxDxDS

DxDxDSD

N

neJ

coshsinh

sinhcosh

coshsinh

sinhcosh 22

0

e

kTD e

e

e

kTD h

h

tB

tT

ehc

IeJ

ehc

IeJ

)(0

)(0

)(

)1()(

1ln1ln

00 B

B

T

T

J

JJ

J

JJ

e

kTV

489

and (65)

The fill factor can then be calculated as a ratio of intensities: .Using these

equations, all relevant solar cell measurements can be simulated.

Simulation Results

Using the equations above, tandem solar cells were simulated using top cell thickness

as the independent variable. The thickness was varied from 10m to 100m. Incident sunlight

intensities, silicon absorption coefficients, and silicon material parameters were obtained from

course lecture notes used by Professor Jain.4 As the top solar cell thickness increased, Voc, Jsc,

and Pmax decreased and the fill factor grew as the additive inverse of Pmax with a plateau at 89%.

Figure 38C shows Pmax as a function of the top cell thickness. At a top cell thickness of 20m,

Voc = 1.19V, Jsc = 18 mA/cm2, and the fill factor was 88%.

Fig. 38C. Output intensity vs. top cell thickness.

Conclusion

The results of the fabricated solar cell were acceptable but left a lot of room for

improvement. Had we been able to make more cells, we probably would have seen slight

improvement in the open circuit voltage and short circuit current. We would also have used a

more convenient top metal contact pattern. This would improve consistency of the results

across the entire surface of the solar cell.

The simulation results were very reasonable but seemed a bit optimistic. The equations

used probably ignore a number of losses that arise during actual fabrication of the solar cells.

This gives slightly higher output intensities and fill factors than we would expect to see in the

lab. However, the simulation still gave reasonable results and showed how those results varied

for different material properties, such as top cell thickness. The program also allows simulation

of semiconductors other than silicon, since the user can input properties of different materials.

This flexibility permits the research group to explore both tandem solar cells and solar cells

made from materials other than silicon.

References1S.R. Kurtz, P. Faine, and J.M. Olson, J. Appl. Phys. 68, 1890 (1990). 2Radio Shack Silicon Solar Cell, Catalog#276-124; http://www.radioshack.com/home/index.jsp

0dJ

dP)( mpmp JVV

scoc JV

PFF max

0

5

10

15

20

25

10 30 50 70 90

Top Solar Cell Thickness (m)

Inte

nsit

y (

mW

/cm

^2)

490

3Silicon Solar Inc. Solar Cell, Catalog #: 04-1193. http://www.siliconsolar.com/#skusearcha 4F. Jain, ECE 245 Supplementary Notes (2006).

6.10.2 Tandem Solar cells using amorphous and crystalline cells

Figure 39 shows a typical tandem cell with amorphous and crystalline solar cells.

Fig. 39. Structure of Tandem solar cell.

The overall voltage current relations are provided below:

Equation 1 shows the total voltage which is a series combination of two voltages in the top

and the bottom cell.

V=VT + VB (66)

The voltage drop across the top and bottom cells are given by

(67)

Alternately, this equation can be written in terms of currents rather than current density J.

(68)

Equation 2B uses the individual solar cell current-voltage equation, which are

expressed below. For the top cell the current is dependent on its reverse saturation current IST

and photon generated current ISCT.

Tunnel Junction

N Amorphous Si,

NDT

P Amorphous

Si, NAT

N Si, NDB

P Si, NAB

Top Ohmic

Contact

Bottom Ohmic Contact

0.015µm

0.5µm

0.5µm

10µm

Top

Amorphous

Si cell

Bottom

Si cell

Homojunction # 1

Top cell

P+ AmorphousSi

n+ Si layer

Current density

J or current I

+

_

Load V

Photons

between

1.5-1.1eV

Photons

greater

than 1.5eV n1

p1

n2

p2

1ln1ln

SB

SCB

ST

SCT

J

JJ

J

JJ

q

kTV

1ln1ln

SB

SCB

ST

SCT

I

II

I

II

q

kTV

491

Eq. 69

Similarly, the bottom cell current is dependent on its voltage VB and short circuit current ISCB.

Eq.70

Here, the reverse saturation current density Js is expressed as:

For the top cell:

JsT= q [(DnT npoT/LnT) + (DpT pnoT/LpT)] (71)

For the bottom cell:

JsB= q [(DnB npoB/LnB) + (DpB pnoB/LpB)] (72)

Because of the finite lengths of the absorbing regions p1 and p2, the boundary

conditions are different and the exact expressions of the reverse saturation current density is

modified [see Section 6.13 ] as Eq.73 below:

𝐽𝑠 = 𝑞𝑛𝑖

2

𝑁𝐴 √

𝐷𝑒

𝜏𝑒(

𝑆𝑒√𝜏𝑒/𝐷𝑒𝑐𝑜𝑠ℎ𝑥(𝑥𝑝/√𝐷𝑒𝜏𝑒) + sinh (𝑥𝑝/√𝐷𝑒𝜏𝑒)

𝑆𝑒√𝜏𝑒/𝐷𝑒𝑠𝑖𝑛ℎ𝑥(𝑥𝑝/√𝐷𝑒𝜏𝑒) + cosh (𝑥𝑝/√𝐷𝑒𝜏𝑒))

+ 𝑞𝑛𝑖

2

𝑁𝐷√

𝐷ℎ

𝜏ℎ(

𝑆𝑒√𝜏𝑒/𝐷𝑒𝑐𝑜𝑠ℎ𝑥(𝑥𝑝/√𝐷𝑒𝜏𝑒) + sinh (𝑥𝑝/√𝐷𝑒𝜏𝑒)

𝑆𝑒√𝜏𝑒/𝐷𝑒𝑠𝑖𝑛ℎ𝑥(𝑥𝑝/√𝐷𝑒𝜏𝑒) + cosh (𝑥𝑝/√𝐷𝑒𝜏𝑒)) (73)

Eq. 73 can be recognized as Eq. 61, if we substitute npo= ni2/NA and Dn/Ln = (Dnn)

1/2.

Here, ni is the intrinsic carrier concentration, NA and ND are the acceptor and donor

concentrations, is the minority carrier lifetime, S is the surface recombination velocity, and x

is the layer thickness. D, the diffusion constants, can be calculated from:

and (74)

where k is Boltzmann’s Constant, T is the temperature, and is the charge carrier mobility.

The short circuit current densities for the top and bottom cells are expressed in Eqs. 66 as:

(75A)

(75B)

where I0() is the incident light intensity as a function of wavelength, h is Planck’s constant, c

is velocity of light, andare the absorption coefficient in top and bottom layers,

respectively, and tpT, tT and tpB are the thicknesses of p-absorbing layer (top cell), total thickness

of top cell, and p-absorbing layer of the bottom cell, respectively. Also, k is the Boltzmann

e

kTD e

e

e

kTD h

h

SCTT

ST IkT

qVII

]1[exp*

SCBB

SB IkT

qVII

]1[exp*

pBBBTT

pTT

tt

SCB

t

SCT

eehc

IqJ

ehc

IqJ

1)/(

)(

)1()/(

)(

)(0

)(0

492

constant, and T the temperature.

The short circuit current of the cell was taken as the lesser of JSCT and JSCB. The open

circuit voltage is calculated by substituting current density J to be zero.

(67)

(76)

Power output per unit area is simply the product of J and V. The maximum output

current density Jmp and voltage Vmp can be obtained using:

.

The fill factor can then be calculated as:

(77)

Using these equations, all relevant solar cell measurements can be simulated.

Results: Single crystalline Si-Solar cell

Thickness Is Isc Voc

tn = 0.5 m,tp = 10 m 7.1752* 10-13 A 0.0456 A 0.6702 V

Fig. 40 I-V Characteristic of Si solar cell.

Single Amorphous Si-Solar cell

Thickness Is Isc Voc

tn = 0.015 m

tp = 0.5 m

2.59* 10-13 A 0.0030 A 0.6256 V

1ln1ln

SB

SCB

ST

SCT

J

JJ

J

JJ

q

kTV

1ln1ln

SB

SCB

ST

SCToc

J

J

J

J

q

kTV

0dJ

dP

scoc JV

PFF max

493

Fig. 41 IV characteristic of Amorphous Si solar cell.

Tandem Solar cell

Thickness Reverse

Saturation

Short circuit

current

density

Tandem

Voc

Max current

density

Jmp

Max output

Vmp

Fill

Factor %

Amorphous Si

(Top) =tT=0.515m

Crystalline Si

(Bottom)

tB= 10.5 m

JST = 2.5923*

10-13 A/cm2

JSB = 7.1752*

10-13 A/cm2

JSCT = 0.0074

A/cm2

JSCB =

0.034A/cm2

For series, we

pick

Jsc =

0.0034A/cm2

1.1989 V 0.0033

A cm-2

1.0866 V 88.5

Tandem solar cell comprising of Amorphous Si and Crystalline Si cell:

Fig. 42 shows the Voltage Current characteristic of the tandem cell. This cell is similar ato

the cell shown in Fig. 44 Sanyo’s Heterojunction with intrinsic thin film (HIT) cell.

494

Fig. 42 I-V characteristic of the tandem solar cell.

6.11 Tandem, MEG, and Quantum Dot Solar Cells: Part-II

6.11.1 Emerging new cell structures:

Next we describe cells which have emerged during last 10 years. These include:

1. Sanyo’s heterojunction with intrinsic thin film (HIT) cell and amorphous Si

window and crystalline Si cell

2. CdTe-CdS cells as commercialized by First Solar Corp.

3. Intermediate Band (IB) Cell: these cells have not been demonstrated.

4. Multiple exciton generation (MEG) cells

5. 4-5 junction tandem cells

6. Organic and polymer cells

7. Quantum dot and quantum wire cells.

6.11.1.1 Fundamentals of Optical Transitions: We need to go over excitonic transitions

before we discuss the new technologies. There are two processes involving photons.

1. Emitting Photons, and

2. Absorbing Photons. The absorption involves transitions involving

a. Excitonic Transitions

b. Free Carrier Transitions

i. Intra band [Within sub bands of well or wire (semiconductors)]

Within valance or conductor (metals)

ii. Inter-band like Valance band to Conduction band

Excitonic transitions take place when not enough energy is given by a photon to get an

electron from the valence band to conduction band or generate an electron and hole pair.

Exciton Binding Energy Eex is related hv= Eg - Eex

hv ≥ Eg hv-Eg =excess energy

495

3. Absorption of photons is relevant not only to solar cells as well as optical

modulators.

We next briefly describe the operation of optical modulators using the above effects..

Optical Modulators: In the case of modulators we harness absorption of photons and

variation of absorption as a funciton of applied external voltage or electric field.

a. Electroabsorption [change in α(Ep)] is used in photodetectors and Optical

modulator

b. Electrorefraction refers to change in the index of refraction nr

(perpendicular electric field) due to an externally applied perpendicular

electric field

Intensity Modulation: Intensity changes due to change in the absorption coefficient

for a travel of distance x in the medium.

I=Ioe-(E) x ⇒ Index of refraction is function of applied field

Phase Modulation: Changes in phase nras light travels in a

medium L due to change in index of refraction nr.

If the Fabry-Perot cavity is made of a medium whose phase can be changed by

applying electric field, we can obtain intensity modulation.

Field-Dependent Birefringence: Polarization of light is function of applied field in

liquid cyrstal displays (LCDs) and multiple quantum wells (MQWs).

Intensity modulation can be obtained if we sandwich a field-dependent birefringent

material between two cross polarizers.

In metals and certain quantum wells/wires structure we observe intra band transitions.

Absorption in quantum well, wires and dots: Photon absorption coefficient increases as we

go from wells to wires and to quantum dots.

Absorption coefficient is related to index of refraction change by Kramer Kronig

relations. Once we know the absorption spectrum , we can find the index of refraction

and changes in its value.

Excitonic Transitions: Quantum Confined Stark Effect in MQWs: Shift of photon

absorption to lower energy is called Stark effect. It involves red shift. See appendix.

Stark effect is used in electroabsorptive modulators using wells, wires and dots.

496

Solar cells: Photon absorption creates e-h pairs in semiconductors. More pairs with

increasing intensity of light. Inter-band absorption is most dominant pathway to absorb

photons. Inter-band absorption is of two types:

1. Direct transitions involving no phonons (see the expression for in Section 6.3)

2. Indirect transitions involving phonons.

Excitonic transitions and multiple exciton generation (MEG) is another recent method to

absorb photons with smaller excess energy loss.

6.11.1.2 Nanostructures in Solar Cells and Current Technologies

Nanostructures are utilized either as wavelength independent absorption layer to minimize

reflection on the solar cells as highly absorbing compact layers forming junction(s)

incorporating quantum dots or wires/tubes.

Solar Cells and Modules: Solar cells can be divided into three broad categories:

1. Inorganic Semiconductor – based solar cells

2. Organic Semiconductor (OSC)- based solar cells

3. Inorganic-Organic hybrid cells

We will primarily focus on inorganic solar cells. Organic Photovoltaics (OPVs) are

briefly discussed at the end of this write up. The material presented here is primarily based on

two monthly volumes of MRS Bulletins (January 2005, and March 2007).

Solar cells are deployed as modules. Modules production and processing depends on

the type of solar cells. It is critical in estimating the cost of energy generation. The chart below

shows the cost of Si based modules (both thin film Si and Si wafers). This has given a

benchmark for other technologies.

497

Fig. 43. Module price as a function of power production.

R. M. Margois, NCVP Solar Program Review Meeting, Denver, CO, 2003.

Ref. S. E. Shaheen et al., Organic based photovoltaics: Toward low-cost power generation, MRS

Bull., p.10, January 2005.

Inorganic solar cells are divided into following categories:

1. Si based cells: (these are also referred to as First Generation). Some of the Si cells,

particularly those including tandem amorphous and microcrystalline cells could be

called 2nd or 3rd generation).

These include Si cells with varying morphology:

Material morphology Conversion efficiency

Single Crystal (SC) Si 24.7% cells,

22.7% modules

Multi-Crystalline Si (mc-Si) or nanocrystalline (nc) 20.3% cells,

15.3% modules

Si Ribbons

2. Thin Films Solar Cells: (also called second generation cells).

Under this category, we will present Si cells and non-Si cells.

(a) Amorphous hydrogenated Si:H and SiGe:H cells are fabricated in two forms. Amorphous-Si:H has an energy gap of 1.7-1.9eV. This means these cells will produce

higher open circuit voltage. However, the photocurrent is smaller due to insufficient

absorption of sun light.

498

Tandem a-Si:H/a-SiGe:H/a-SiGe: H cells have demonstrated very high efficiency.

Some examples are given below: (R. Schropp et al., MRS Bull., p.219, March 2007).

Table: Amorphous and Poly-crystalline/Micro or nanocrystalline cells

Material Efficiency #of Junctions Company

a-Si/a-SiGe/nc-Si:H 15.07% 3 United

Solar Ovonic

c-Si:H 10.3% 1 IPV Julich

SPC Poly-Si 9.8% 1 CSG Solar

(SPC=Solid phase crystallization; AlC=Aluminun induced crystallization)

6.11.2 Heterojunction with intrinsic thin film (HIT) cell

(Ref: M. Tanaka et al, Photovoltaic Energy Conversion, Proc. 3rd word Conf, 1, p.955, 2003.

It consists of c-Si layer with a-Si (p-n) junction on top (having transparent conducting oxide

TCO and a metallic collection grid contact). HIT cell based modules have reported an

efficiency of 19.3% by Sanyo.

Fig. 44 Sanyo’s

Heterojunction with intrinsic

thin film (HIT) cell.

(a) cSi-aSi-TCO-Metal grid

contact HIT n-aSiH/a-

n+Si/p-type Si wafer.

(b) Glass/TCO(bottom

contact)/p-cSi (30nm)/i-

cSi(1m)/n-aSi

(40nm)/ZnO/Ag/Al

TCO= Transparent

conducting oxides serving as

contact. tin oxide, indium

tin oxide, ZnO.

Fig. 45. Module cost /Wp in 2005. [Ref: A. Slaoui and R. Collins, p. 211, MRS Bull, March 07].

The goal here is to obtain $1.82 to $1.2/Wp in 2010 and 2015, respectively.

499

6.11.3 Thin film CdSe and CuInSe2 cells

Ref: J. Beach and B. McCandless (p.225, March 2007 MRS Bull). This

category includes cells made out of II-VI and chalcopyrite. Typical cells are:

Copper Indium diselenide (CIS), 2.CdTe, and 3. Cu(InGa)Se2 (CIGS) CdTe-

CuInSe2 cells are sensitive to moisture. Glass sheets are used to seal front and

back and polymers are used to seal the edges and bond glass. For AM 1.5

Solar Spectrum is: see http://rredc.nrel.gov/solar/spectra/am1.5/

Fig. 46. Solar spectrum under AM1.5 (To convert nanometers to µm, divide by 1000. To convert

W/sm/nm to W/sm/µm, multiply by 1000.)

First Solar disclosed cost of CdTe/CdS cell for a 25MW peak/year (MWp/year)

was $1.59/Wp. This is based on a module efficiency of 9%.

Cost goals: The DOE goal Solar America Initiative aims at $0.15 per kWh, and

system cost of $2/Wp by 2015. This generally means that the module cost should

be $1/Wp.

Lower Generation Cost/Watt requires:

Higher cell conversion efficiency

Cell efficiency resulting in module efficiency and module cost

High Throughput <-> Reduced manufacturing cost and system cost

p-CdTe/n-CdS cells are η= 16.5%. The alternative being studied includes a-

CdSxOy cells. Doping of CdTe is due to vacancies. CdTe becomes n type with

Te vacancies and p type with Cd vacancies.

Deposition of CdTe is followed by processing including annealing at temperature

T> 4000C in the presence of (CdCl2 + Oxygen) which gives optimal efficiency.

CdTe films grown at low temperature becomes recrystallized during this step.

500

This improves the carrier life time which approaches n~2ns. These cells give an

open circuit voltage of Voc~810mV.

A comparison of CdS/CdTe cell with CIGS cell is shown below.

CdS/CdTe => 1.5eV (band gap Eg) CIGS cell CdS/Cu(InGa)Se2/Mo

contact on glass

Voc= 845 mV 679mV

Jsc= 25.9 mA/cm2 35.1

FF= 75.5% 79.6

In CIGS small cell the efficiency is 19.5% and in a module it is 10%.

The bandgap of CuGaSe2 is 1.7eV and for CuInSe2 is 1.0eV. Others can be

extrapolated.

In a Cu(InGa)Se2 cell, with band gap of 1.67eV, the conversion efficiency was 9.5%.

Ref: R. Mickelsen et al, 15th IEEE PVSC conference, p. 800, 1981. These values have

significantly improved during the past 5 years.

Processing of CdTe-CdS cells on flexible glass substrates: There are various

processing steps for the cell realized on a flexible willow glass substrate Fig. 47:

TiZnTe: CuCdTe (4-5µm)

CdSTO (Tin Oxide)

}500nm

Glass Substrate

(Corning Willow Glass)

Fig. 47 Cross-sectional schematic of CdTe-CdS cell on flexible Willow (Corning) glass

substrate. [ref. Rance et al (2014) have produced 14.5% efficiency]

1. Deposit fluorine doped tin oxide (FTO) and tin oxide (TO) using MOCVD at 550°C.

The sheet resistance is 20 Ohm/square.

2. Deposit CdS thin film in a bath using Cd acetate, thoiurea, ammonium acetate, and

ammonium hydroxide at 92°C.

3. Deposit 4-5 micron CdTe film using closed space sublimation (CSS) keeping glass

substrate at 600°C using CdTe source plate at 660°C for 2 minutes.

4. CdTe layer is vapor treated with CdCl2 using CSS at 400°C for 10 minutes.

5. Back Ohmic contact to CdTe layer following heat treatment. First a 100nm layer of

CdTe is removed using Argon ion milling. This is followed by the deposition of ZnTe

layer doped with 2% Cu via RF sputtering. A Ti layer is DC sputtered at 300-330°C.

The total thickness of the Ohmic contact is 500nm (Zn Te: Cu + Ti = 500nm).

6. Cells are isolated by removing Ti film using Transenee TFT etchant. ZnTe film is

removed using 35% FeCl3 in water.

Processing of CdTe-CdS cells on polyimide (PI) substrates

501

The UPILEX polyimide sheet 7.5 micron in thickness was deposited with transparent

conductive oxide (TCO) including tin doped indium tin oxide (ITO) and aluminum doped

ITO. A layer of ZnO is deposited to enhance the reproducibility of the process of solar cell

fabrication. The ITO and intrinsic ZnO are deposited by rf-sputtering at 300°C. CdS thin

film is deposited using high vacuum thermal evaporation at 165°C. This is followed by

CdTe evaporation at 350°C.

A layer of CdCl2 is thermally evaporated (20-640nm) on the CdTe surface and annealed at

420°C for 20 minutes. After the etching of CdTe top layer using bromine and methanol

solution, topcontacts of Cu/Au or pentacene/metal were deposited.

A conversion efficiency of 10% under AM1.5 conditions was reported for the polyimide

cells FF=0.678, Jsc = 18.2 mA/cm2, Voc= 813 mV. The results were better (12%

efficiency) for soda lime glass (SLG) substrate.

CdCl2 deposition &

Anneal at 420°C

CdTe 350°C

75µm

Polyimide in Superstate UPILEX

Au

Pentacene high

work function

Tin doped ITOAl doped ITO

ZnO

TCO

Fig. 48 CdS-CdTe cell on flexible polyimide thin film.

Ref. Perrenoud, S. Buecheler, A.N. Tiwari, “Flexible CdTe Solar Cells with High

Photovoltaic Conversion Efficiency”, 2009 IEEE Conference, p.695-699, 2009.

Question: Why Voc/Eg ratio is reduced in cells having absorber semiconductor band gap

greater than 1.3eV.

A typical CdInGaSe2 (CIGS) cell is fabricated on a Molybdenum (Mo) coated soda lime

glass substrate. Over the Mo layer an absorbing layer of CIGS is deposited. This is followed

by the deposition of n-CdS. N-CdS is treated in various proprietary ways to improve the cell

characteristics. Finally transparent conducing oxides are grown. In the CIGS reported above,

ZnO and ITO conducting transparent oxides were deposited.

6.11.4 Multi-junction or Tandem Solar Cell 3rd Generation

(Ref: Dimroth and S. Kurtz, MRS Bull, March 2007, p. 230). These cells have multiple p-n

homo or heterojunctions in series, separated by one or more tunnel junctions. Expensive single

crystal substrates are used and these cells are generally operated under concentrated sun light

conditions. One sun is 1kw/m2. See Figure 49 which shows top and bottom cells connected by

a tunnel junction. (Ref: R.A Metzger, Manufacturing III-V Solar cells for space Application,

page 25, Compound Semiconductor November / December 1996).

502

Fig. 49 Cross-section of a two junction tandem cell (S. Kurtz et al., NREL). Same as Fig. 1.

Other triple junction cells are shown below. The key is to absorb part of the photons in an energy

band gap descending order. Start with a wider Eg and go to lowest Eg. Ge is lowest at 0.67eV.

F.Dimroth and S. Kurtz (p.230, MRS Bull., March 2007) have shown this in three figures

below. Fig. 50 Energy gap as a function of lattice constant in various semiconductors.

Fig. 50 Energy

gap as a

function of

lattice constant

in various

semiconductors.

GaAs Substrate Zn-doped

GaInP 0.07um p=3*10^17 cm-3 [Zn]

GaAs 3.5um p=8*10^16 cm-3 [Zn]

GaAs 0.1um n=1*10^18 cm-3 [Se]

GaInP 0.1um n=1*10^18 cm-3 [Se]

GaAs 0.01um n=1*10^19 cm-3 [Se]

GaAs 0.01um p=8*10^19 cm-3 [C]

GaInP 0.6um p=1.5*10^17 cm-3 [Zn]

GaInP 0.1um n=2*10^18 cm-3 [Se]

AlInP 0.025um n=4*10^17 cm-3 [Si]

GaAs 0.5um

n=6*10^18cm-3 [se]

Au

3um AR Coat (2 layer)

Front Grids

Contacting layers

(Eg=1.86ev)

Back Contact

Top Cell

Tunnel

Cell

Bottom Cell

GaInP/GaAs Tandem Cell

503

The absorption in various semiconductors for AM1.5 is shown below in Fig. 51..

Fig. 51 (a) Energy used by a single junction Si cell under AM1.5. Fig. 51 (b) Energy used by a

single junction Ge, GaInAs (Eg=1.18eV) and GaInP (1.7eV) cells under AM1.5 conditions.

Figure 52 shows the theoretical efficiencies for various systems. (d) and (e) show

mismatched systems using metamorphic layer (dark layer).

Fig. 52. Various material systems on Ge substrates.

Reported dual Junction GaInP/GaAs cell is 39% under 240 suns. The material is

Ga0.5In0.5As (1.7eV)/Ga0.9In0.01As(1.18eV)/Ge (0.67eV) substrate.

Single Junction Double Triple Sun Concentration (X)

% 25% 29% 31% 1

31% 39% 240

Ref: F. Dimroth, Phys. Status Solidi, 3, (3) p.373 (2006).

504

The offset [(Eg/e) – Voc] is an indicator of nonradiative losses in multi-junction

solar cells. Some values of this offset are given below.

Table Loss due to Offset Voltage [Voc/(Eg/q)]

Semiconductor Band gap (eV) Offset Volt [(Eg/q) – Voc]

GaAs 1.44 0.387

Ga0.83In0.17As 1.118 0.390

GaInNAs 1.07 0.600

Ref:R. King et al., 20th Proc. Ear. Photovol. Solar Energy Conf., p118, 2005 (Barcelona,

Spain).

Companies involved in the development are: Spectra Lab, Emcore, AZUR Space

Solar Power, and others.

Device Structure for a Triple Junction Cell (structure e): This cell (shown

below) produced a conversion efficiency of 37.9% at 10 Suns and 40% is

expected for higher concentrations. Ref: A. Bett. et al., Proc. WCPEC-4

(Waikoloa, Hawaii) 2006, p.729.

External Quantum efficiency (EQE) is a measure of carrier collection efficiency

of P-N junction. Lattice-matched and mismatched materials are used in double

and triple junction cells. Some examples are given below. In the case of

mismatched materials, metamorphic scheme is employed.

Projected conversion efficiencies are for various 3rd generation cells:

I. 66% Three-junction tandem cells

40-50% in some Multi-junction or Tandem Cells

II 66% Quantum Dot solar cells using Multi-Exciton generation (MEG)

III. Intermediate band (IB) devices.

IV. Si nanowire solar cells.

6.11.5 Multiple exciton generation (MEG), IB cells, and Quantum Dot based solar cells

When the photon energy h is greater than the bandgap, the electron and hole

generated have excess energy (h-Eg) that is given up as phonons eventually heating up

the lattice. That is, the energy of the hot carriers is lost.

Excess energy can be recovered in following ways: (a)recover hot carriers

before they thermalize (ref 2-4), and (b) hot carriers producing 2-4 electron hole pair

via impact ionization or via multiple exciton generation. Multiple exciton generation

(MEG, this is shown in QDs of PbSe, CdSe etc.)

Impact Ionization

a. 1 Photon creates 2 electron-hole pairs or exciton pairs. This is known as MEG.

b. A new possible mechanism for MEG involves simultaneous creation of

multiple excitons.

505

No group has yet reported enhanced photo carriers in the external circuit.

Quantum Yield in 300%. 3 Exciton are released.

Types of QD cells:

1. Photo electrodes composed of QD arrays,

2. QDs such as InP, 3-5nm used to sensitize a TiO2 (30nm) nanocrystalline film, and

3. QDs dispersed in organic semiconductors in blend of electron and hole

transporting polymers.

Additional details in following three pages.

New Mechanisms: use of QDs

Problem: Loss of Hot Carrier Energy (h-Eg)

1. MEG Multiple Exciton Generation: A new method to recover hot carrier energy.

(in quantum dots)

Excess Energy:

Recover Hot carriers before they Thermalize (ref-2-4).

Hot Carriers Producing 2-4 Electron Hole Pairs.

QDs: PbSe, PbS, Pbte, CdSe

Quantum Dot diameter Effective Eg

PbSe 5-7 nm 0.73 eV

4-7 nm 0.82

3.9 nm 0.91

h/Eg = 4.0

Impact Ionization

a. 1 Photon creates 2 electron–hole pairs. This is known as MEG. The electron-

hole pair when bound is called exciton. In low dielectric constant materials

excitons form readily.

b. A new possible mechanism for MEG involves simultaneous creation of multiple

excitons.

506

Fig. 53 Quantum yield is 300% if 3 excitons are formed.

No group has yet reported enhanced photocurrent in an external circuit due to this

MEG effect.

2. Intermediate Band (IB): IB cells reduce the loss of below band gap and sub-band

gap photons. Here an intermediate level is introduced by introducing a deep impurity

level in the band gap. This is shown theoretically to be useful for single junction as

well as tandem cells. Quantum efficiencies between 1 (or 100%) and 2 are predicted.

Ref. A. Luque et al, Phys. Rev. Lett. 78, p. 5014 (1997).

Fig. 52 shows Intermediate band (due to impurity level or levels; superlattice mini-

bands, lone pair bands) in the middle of CB and VB.

Fig. 54. Intermediate band schematic.

IB cells reduce loss due to long wavelength photons. See also Fig. 60.

3. Types of Quantum Dot based Cells

507

a) Photoelectrodes composed of QD arrays.

b) QDs (such as InP, 3-5nm dia) used to sensitize a TiO2 nanocrystalline film.

c) QDs dispersed in Organic/polymeric Semiconductors [i.e. in blend of electron and

hole transporting polymers].

1. Photoelectrodes composed of QD arrays

Fig. 55. 3-D Analog of mini-bands found in superlattices.

2. TCO (transparent conducing oxide) deposited with InP-sensitized TiO2 (30nm)

qc

e

h

n nn

n n

nn n

nTCO Poly

mer

Vout

4.1eV

Eg

Hole Transporting

Polymer 1.8eV-2.6eV

QD

InP

Microsphere

TiO2

q

Hole

Vacuum level

InP qc

Incident

photons

+-Incident

photons

Fig. 56. TiO2 microspheres functionalized with InP quantum dots with a hole transporting

layer.

3. QDs dispersed in Organic/polymeric Semiconductors (i.e. in blend of electron and

hole transporting polymers).

New Mechanisms: use of nanowires [Th. Stelzner et al, Si nanowire-based solar cells,

Nanotechnology, 19, 295203 (2008).]

6.11.6 Organic Photovoltaics (OPVs)

1. Materials are inexpensive

2. High absorption coefficient 100nm thin films

3. Fabrication process is high –throughput; low temperature

508

4. Roll to roll.

5. Flexibility in designing molecular orbital’s, doping, organic-organic, organic-

inorganic. If efficiencies are comparable or even slightly lower than existing

technologies, OPV make a compelling case.

Limitations:

1. Stability of organic materials (organic LED exhibit reasonable life time)

2. Device degradation due to change in morphology, film adhesion

3. Exciton decay length 110nm

Organic semiconductors: Small molecules in between polymers.

Molecular orbital’s built form individual extonic orbital. Device processing depends

small molecular.

Light absorption in OSC result in formation of excitons. Excitons dissociate at or by

1. High electric field

2. Defects

3. Interface band offset

Excitons diffuse at an interface between two materials electron transfer for donor material

at acceptor material or hole transfer from acceptor to donor.

A typical OSC photovoltaic cell Exciton decay length ~ 10 nm

Arrangements of molecular in an OSC is disordered. Transport of charge is by hopping

between molecular. Crystalline OSC exhibit higher μ.

Approaches used in OPV’s

Small molecular OSCs- tandem cell.

Organic Photovoltaics:

Materials are inexpensive

High absorptive coefficient 100nm thin films

Fabrication process is high- throughput, low temperature

Roll to roll

Flexibility process in designing molecular orbitals,doping,organic-ozone, if

efficiies are comparable or even slightly lower than existing technologies,OPVs

make a compelling case.

Limitations:

Stability of organic materials (organic LEDs exhibit reasonable life time)

Device degradation due to change in morphology, film adhesion, interdiffusion of

species.

Excitons decay length 10nm (from an interpreter); limiting optical density and

power consumption.

509

Organic semiconductors: OSC’s are divided into

Small molecule

Dendrimers

Polymers

Device processing depends on if it uses small molecules or polymers.

Conduction band is represented by lowest unoccupied molecular orbitals, LUMO

and valence band is equivalent to HOMO or highest occupied molecular orbitals.

Light absorption in OSCs results in the formation of excitons.

Excitons disassociate by

o High electric field

o Defects

o Interface band offset.

o Electron transfer from donor material (or electron transporting layers) to

acceptor material or hole transporting layer.

A typical exciton decay length is10nm. As a result, these cells are very thin.

Organic Solar Cells (OSC) schematic.

1% conversion efficiency

Electrode

Electron transporting

or Donor like

Negative Electrode

Junction

Substrate

Fig. 57. OSC schematic.

Arrangement of molecules in an OSC is disordered (like amorphous). Transport of

charges is by happing between molecules

OSC µ => 10−6 cm2 V-1 S-1 to 10-3 cm2 V-1S-1

Crystalline OSC exhibit higher µ. In the anthracene and pentacene based cells the current

transport model is similar to inorganic semiconductor band transport model.

New Mechanisms: Use of Quantum Dots (QDs)

Problem: Loss of Hot Carrier Energy (h-Eg)

Multiple Exciton Generation (MEG): A new method to recover the hot carrier energy

in quantum dots.

510

Excess Energy: Recover Hot carriers before they thermalize

Hot carriers producing 2-4 electron hole pairs.

Quantum dot materials:

PbSe (Lead Selenide)

PbS (Lead Sulfide)

PbTe (Lead Telluride)

CdSe (Cadmium Selenide)

Example: PbSe (Lead Selenide)

Quantum Dot Diameter Effective Eg

5-7nm 0.73eV

4-7nm 0.82eV

3.9nm 0.91eV

h/Eg=4.0

h1 h2

Effective Gap

and Photon

absorption

Fig. 58. Quantum confinement schematic.

Impact Ionization:

Process where an electron moves a valence band electron out into the conduction

band forming an electron hole-pair.

1 Photon creates 2 electron- hole pairs. This is known as MEG. The electron-hole pair

when bound is called an exciton. In low dielectric materials, excitons form readily.

A new possible mechanism for MEG involves simultaneous creation of multiple

excitons.

511

Fig. 59. Quantum yield is 300% if 3 excitons are formed. No group has yet reported enhanced

photocurrent in an external circuit due to this MEG effect.

Intermediate Band (IB): Loss of below band gap and sub-band gap photons. Here an

intermediate level if introduced by introducing a deep impurity level in the band gap. This

is shown to theoretically useful for single junction and tandem cells. Quantum efficiency

between 1(100%) and 2 are predicted.

Fig: 60 Intermediate band [due to

impurity level or levels; superlattice

mini-bands (See below) lone pair bands]

is shown in between the conduction

band and valence band.

To avoid loss due to longer wavelength

photon: Ge=0.67eV

-hv<.67eV

-hv<1.1eV in Si(silicone)

Types of Quantum Dot (QD) based Cells:

a. Photoelectrodes composed of QD Arrays.

b. QDs (such as InP, 3-5nm dia) used to sensitize a TiO2 nanocrystalline film.

512

c. QD’s dispersed in Organic/ polymeric Semiconductors (i.e. in blend of

electron and hole transport polymers).

Photoelectrodes composed of QD arrays:

QD Layers having different sizes to absorb different part of the solar spectrum.

QD1 QD2 QD3

CB

QD Layersn p

Conduction band energy level

for various QD layers

Fig: 61 3-Dimensional analog of mini-bands formed in superlattices.

Ref : A. Luque et al, Phys. Rev. Lett. 78, p.5014 (1997)

Quantum Dots dispersed in Organic/polymeric Semiconductors. TCO (transparent conducing oxide) are deposited with InP-sensitized TiO2 (30nm). In

addition there is a blend of electron and hole-transporting polymers. This is shown in Fig.

62.

513

QD

CdSe quantum dots and Nanorods

˗ Nano dots must be cladded

Vac

Polymer Electron Transporting

LUMO

HOMO

EzEᵪ

Fig .62 QDs in polymer matrix.

Ref: New Mechanisms: Use of nanowires [Th. Stelzner et al, Si nanowire-based solar cells,

Nanotechnology, 19, 295203 (2008)].

514

6.12 Problem set and Solutions for Solar Cell Design and Tandem Cells Q.1 Solar Cell Design: Design an n+-p Si solar cell for air mass m=1 (AM1) or AM1.5.

Assume that the incident radiation for AM1 (Fig. 1) is 92.5mW/cm2. IL=ISC = 38.97mA/cm2.

Given: p-Si crystalline wafer with doping of 2x1017cm-3 or resistivity of ~0.1 Ohm-cm.

n+-side: Donor concentration ND=1020 cm-3, minority hole lifetime p=2x10-6 sec.

Minority hole diffusion coefficient Dp=12.5 cm2/sec.

p-side: Acceptor concentration NA= 2x1017cm-3, n=10-5sec. Dn=40 cm2/sec. Junction

area A=1 cm-2, ni (at 300K) = 1.5x1010cm-3, r (Si)=11.8, 0=8.85x10-14 F/cm, =0r.

Assume all donors and acceptors to be ionized at T=300°K.

a) Find the surface reflection loss. Find the thickness and index of refraction of the AR

coating material (identify the wavelength you are using in your design). Other

approaches to AR coating: Surface texturing to prevent surface reflection? You may

want to do a quick literature search. (Do not spend too much time on it).

b) What are the thicknesses of the p-Si and n+ Si layers?

c) Determine the maximum power point Pm = Vm Im and the fill factor (FF).

d) Compute dominant losses:

i) Long wavelength photons h<Eg (=1.1eV).

ii) Excess photon energy not used in generating electron-hole pairs.

iii) FF

iv) Voltage factor.

Show after all the above losses (see worked out solar design in the Notes pp.

431-443) that the cell could be 12% efficient.

a) Why an n+-n/p-p+ cell structure is better than the n+-p Si cell? Briefly show such

a structure.

HINT: Follow design example pp. 431-443 for above parts. See also pp.479.

0.2 2.62.01.40.8

200

0

2200

2000

1800

1600

1400

1200

1000

800

600

400

1.1 1.7 2.3

Wavelength (m)

Sp

ectr

al

Irra

dia

nce

(W

/m-2

m-1

)

2400

Air Mass Zero (1353 W/

m2)

Air Mass One (925 W/m2)

Visible

0.31

GaAs (c = 0.87 m)

Si (c = 1.1 m)

1

2

3

4

0.51 0.71

h1.11.3

A

B

C

J

H

I

E

F

DG

Fig.1 Solar spectrum at m=0 and m=1. (Fig. 31 from page 440 of Notes PT-II).

Q.2. For the tandem cell of Fig. 2:

515

b) Find the solar power that is converted by the top cell and power that is available to

the bottom cell.

c) Find the open circuit voltage Voc and Vmp and Imp for each cell.

d) Find the operating current and voltage for the total tandem cell comprising of an

amorphous a-Si n-p junction cell (top) and a crystalline n-p junction cell (bottom).

Assume the voltage drop across tunnel junction=0.03V. Briefly sketch the tunnel

junction. HINT: Pages 452 to 459. No big eqns.

Given:

Short circuit current in the two cells is 7 mA for air mass 1.

The top a-Si cells converts mostly photons having energy h>1.5eV and the bottom

cell converts between 1.5eV> h>1.1eV and any energy not absorbed by the top cell

(for h>1.5eV).

Doping of a-Si cell is similar to that for c-Si cell. (4) n=10-5sec Dn=0.80 cm2/sec in p

type a-Si and Dp=0.25 cm2/sec and p=2x10-6 sec in n-type a-Si layer.

Tunnel Junction

N Amorphous Si,

NDT

p- amorphous

Si, NAT

N Si, NDB

P Si, NAB

Top Ohmic

Contact

Bottom Ohmic Contact

0.015µm

0.5µm

0.1µm

50µm

Top a-Si cell

Bottom

c-Si cell

Homojunction # 1

Top cell

p+ a-Si (0.01m)

n+ Si layer (0.01m)

Current I

+

_

Load V

Photons

between

1.5-1.1eV

Photons > 1.5eV

n1

p1

n2

p2

Fig. 2. Amorphous Si (top) and crystalline Si

(bottom) tandem cell.

Fig. 3. The absorption coefficient in c-Si and a-Si.

Optional

Q.3. What is the advantage of a Quantum dot solar cell?

516

Solution Set: Question 1 Solar Cell Design Show after all losses (excess energy loss, long

wavelength photons below Eg of Si, voltage factor gOC EV / , and fill factor and surface

reflection) the Si cell could be 12% efficient. (Design for A=1 cm2).

Contact stripe

AR coating

Junction

Back Contact

N+-side p-side

20

2142

56

317310320

1300

sec/90 /10*85.8 sec/5.12

sec10 8.11 sec10*2

10*2 10*5.1 10

cmAreaKT

cmDcmFcmD

cmNcmncmN

nop

nrp

AiD

For AM1 condition, the incident solar spectrum generates a short circuit current of 2

SC 38.57mA/cmI in a cell of 1cm2 area.

a) Surface reflection loss

Index of refraction of Si is 435.38.11,, sirSirn

Reflectivity of Si= 3.01435.3

1435.322

airr

airr

nn

nnR

Reflection Loss=Pin*0.3=27.75mW/cm2 for 2/5.921 cmmWPAM in or 925W/m2

Antireflection Coatings (AR) can mitigate the reflection loss.

Solar spectrum is very broad. We design a coating of thickness‘t’ with an index of refraction

nr2 to eliminate reflection loss.

AR Coating

nr2

t n-Si= 0.5m Junction

For NA=2*1017cm-3, = 0.1 Ohm-cm

Schematic of solar cell

p-Si

p-Si

d

cmandnr 1.0,435.3

n+-Si

517

Å5.863,0863.0853.1*4

64.01

4

64.0,1 ,12

4

853.1435.3* =coating AR ofIndex

9375.1 64.0

2

2

2

AR

r

avr

rairr

avav

ortmm

n

mandtlltn

nnn

eVhorm

(b) Determine the thickness of n+ and p-Si layers.

The thickness of n+ layer is as thin as possible. The only consideration is that series

resistance Rs loss (voltage drop and power dissipation) is very small. That is Rs* Isc <

0.05V. IL=ISC = 38.97mA/cm2.

Rs < 0.05/38.97*10-3 or, Rs < 1.25 Ohm.

Rs is related on resistivity of the n+ layer (which depends on the doping

concentration ND; for 1020cm-3, it is 8x10-4 Ohm-cm) ) and it’s thickness t

(unknown). Here, we are neglecting the drop across p-Si thick layer d with =0.1

Ohm-cm).

Rs = L(A), A is the area of cross-section (=t*cell length 1cm) and length L is the

separation between top Ohmic contact lines. Here, t (n-Si) =L/Rs

=0.1*8x10-4/(1) = 0.8 microns. We have selected 0.5

microns which is true if the separation is less than 1mm.

The thickness of p-Si is determined by the absorption length of photons which is 2/

or 3/ where is the absorption coefficient. Another criterion to determine the

thickness d of p-Si is to keep it 2-3 diffusion length of minority electrons 2Ln-3Ln. Ln

is 2x10-2 cm so 2 Ln = 4x10-2cm or 400 microns. Thickness may be small using 2/.

(c) Determine the maximum power point Vm, Im

Vm and Im are expressed as: .

SCkT

qV

Sm

m

OCm IeIIkT

qV

q

kTVV

m

1 and 1ln

Here,

S

SSCOC

I

II

q

kTV ln .

We need to find VOC to find Vm, and to find VOC we need to find the reverse saturation

current Is as ISC is given to be 38.97 mA.

Is is expressed as

n

pon

p

nop

L

nD

L

PDAq ** =Is

p-side:

cmDL

cmxN

nn

nnn

A

i

po

25

33

17

2102

10*210*40

10125.110*2

10*5.1

518

n+-side: (window region)

3

20

2102

25.210

10*5.1 cmN

np

D

ino

𝐿𝑝 = √𝐷𝑝𝜏𝑝 = √2 ∗ 10−6 ∗ 12.5 = 5 ∗ 10−3𝑐𝑚

𝐼𝑠 = 𝑞𝐴 [𝐷𝑛𝑛𝑝𝑜

𝐿𝑛+

𝐷𝑝𝑃𝑛𝑜

𝐿𝑝]

= 1.6 ∗ 10−19 ∗ 1 ∗ [40 ∗ 1.125 ∗ 103

2 ∗ 10−2+

12.5 ∗ 2.25

5 ∗ 10−3]

= 1.6 ∗ 10−19 ∗ 1 ∗ [40 ∗ 1.125 ∗ 103

2 ∗ 10−2+ 2.5 ∗ 2.5 ∗ 103]

= 0.361 ∗ 10−12A

S

SSCOC

I

II

q

kTV ln

=0.0259 ln[(38.97x10-3 + 0.361x10-12)/ 0.361x10-12]

VOC = 0.6579Volt

SCkT

qV

Smm

OCm IeIIkT

qV

q

kTVV

m

1 and 1ln

Substitute VOC and kT/q = 0.0259V in 1ln

kT

qV

q

kTVV m

OCm

Vm =

0259.01ln0259.06579.0 mV

. Since Vm is on both sides, we need to

write a short program or do trial and error substitution. We know Vm is less than

VOC, so we guess it to be 0.5V. for this guess we tabulate Left Hand Side (LHS)

and Right Hand Side (RHS) until the value of Vm makes both sides equal.

LHS RHS

0.5V(first guess) 0.5799

0.57 0.5766

0.576 0.5764

0.5765V 0.5764

So Vm = 0.5765V and Im is obtained by substituting Vm in the current equation.

Im = 0.36X10-12[exp(0.5765/0.0259) -1] -38.97X10-3

=1.675x10-3 -38.97x10-3 = -37.29 mA

519

Two curves related to solar spectral irradiance

(d) Compute dominant losses

(i) Reflection has been computed before.

(ii) We now compute the long wavelength photons that are not absorbed. These are

photons below the energy gap Eg = 1.1eV for Si. These are shown in the above Figure, in

area of region 1, 2 & 3, and a triangle F’/1.1 (D) /1.16 (D’) micron point on x-axis.

The solar power in these regions is:

Region #1 68.4 W/m2

Region #2 72.69 W/m2

Region #3 35.77 W/m2

These values have been calculated in solar design before.

Region #F’DD’ power is 16.5 W/m2 as shown below.

2'' /W5.1606.0*550*2

11.116.1*550*

2

1**

2

1mDDDFArea

Total long wavelength photon loss is 193.36 W/m2 or 19.36 mW/cm2.

The % of long wavelength light loss at AM1 in Si = %20925

36.193

(iii) We next calculate the Excess Energy Loss for photons above Eg or 1.1eV which have

more energy than needed to create an electron-hole pair.

520

Solar spectral irradiance for AM1

Excess photon energy not utilized in electron hole pair generation Si g,E-h =

Let AM1 plot above eV 1.1 hor m1.1 is divided in several regions. The exact way

is to find the area under the curve numerically. Regions are: Triangle 𝛼𝛽𝐽, Trapezoid

𝐽𝛽𝛾𝛿, Trapezoid marked as #4. These are calculated next.

Excess photon energy lost in spectral region represented by triangle 𝛼𝛽𝐽

eV11.21.121.3energyphoton Excess

12.32

43.24h

eV43.20.51

1.24at energy Photon

eV40.31

1.24at energy Photon J

eV

J

ave

Area if the 2/155

2

31.051.0*1550

2* mW

JJJ

Excess energy not used2/10111.2*

21.3

155mW

521

eVh

eVJatheVJathj

953.12

43.2476.1

43.251.0

24.1 ,43.2

51.0

24.1 Trapezoid

ave

2/5.4122/9501550*51.084.051.084.0*950 RectangleTrapezoid mWJ

Excess energy lost 2/05.178843.0*953.1/5.412 mW

Trapezoid #4, Rectangle #5 & Rectangle #6 can be combined by a rectangle

2/1301.184.0*500 mW

eVh ave 3.12

127.1476.1

2

1.1

24.1

84.0

24.1

Excess energy = eVEh gave 19.01.13.1

Excess energy not used =2/1919.0*

3.1

130mW

eV

Total excess energy loss = 101 + 178.05 + 19 = 298.05 W/m2.

(iv) Voltage factor is defined as the ratio of VOC and Eg/q.

Voltage factor =𝑉𝑜𝑐

𝐸𝑔/𝑞=0.6579/1.1 = 0.598.

The loss is 433.59 x 0.598 = 259.28W/m2, and the % loss is 259.28/925 = 28.03%. This

leaves available power of 174.31 W/m2.

(v) Fill Factor (FF) is defined as (VmIm)/(VOC ISC).

FF = [0.5765*37.29mA]/[0.6579*38.97mA] = 0.834 ~ 0.84.

The power loss due to FF is 0.84*174.31=27.89 W/m2. The % loss is 27.89/925 = 0.03%.

(vi) Collection efficiency of photo-generated electrons and holes. In crystalline Si this

loss may be 10%. However, it is significant in poly-crystalline Si substrates, where it is

~20%. A 10% loss reduces the available power to (146.42 – 14.64) 131.78 W/m2.

(vii) Series resistance loss: Series resistance loss Rs and other losses (see Fig. 19b).

100%

0%

18.9%

925 W/m2

731.64 W/m2

433.59 W/m2

46.93%

79.1%

146.42 W/m2

32.2% Loss

Fill Factor=0.84, Loss=27,89W/m2

Excess Energy (hѴave-Eg) loss, 298.05

193.36W/m2

Power

20.9% Loss Long Wavelength Loss

% Efficiency

Voltage Factor28.03%

259.28 W/m2 Loss 174.31 W/m

2

18.87%

Losses in Si solar cell.

522

2) Question 2

a) Power converted by the top cell

Incident power =ΔαßJ + Trapezoid ßγ'δ'J = ΔαßJ + Trapezoid ßγ'δ'J +γγ'δ'δ

Form Q.1 of this solution the area of various spectral regions are:

1- ΔαßJ =155 W/m²

2- Trapezoid ßγ'δ'J =412.5 W/m²

3- γγ'δ'δ =1010*(0.826-0.8) ; where δ'=0.826 , δ=0.8

=26.26 W/m²

Total power incident =593.76 W/m²

Assuming AR coating at top cell a-Si. The absorption coefficient αav ≈ 2*104 cm-1

Power transmitted = * 10*2*10*.015)-(0.5 4-4

e 593.76

= 593.76 e-0.615*2

= 593.76 e-1.230

= 173.53 W/m²

Power absorbed =593.76 - 173.53 = 420.22 W/m²

Power available to bottom cell = 173.53 W/m²

b) Top Cell Calculations: Find Voc, Vmp, Imp for top cell. In reality, this needs to be

calculated from power absorbed. Isc=7 mA is given.

)ln(s

sSC

I

II

q

kTVoc

Is for an amorphous a-Si cell can be calculated using ni value for a-Si. The ni is

proportional to exp(-Eg/kT). Although the effective masses are different, we get a ball

park value using the following.

p

nop

n

pon

s

aSiiaSiiaSii

L

pDAq

L

nDAqI

cmnenecSinin

******

10*64.6,*10*5.1|,*)(|

Dominates

36)

0259.0*2

4.0(

10)

0259.0*2

1.15.1(

Dn=0.8 cm2/s , τn=10-5 s ;Ln=(Dn* τn)1/2 = (8*10-6) 1/2=2.82*10-3 cm

AmpI

cmn

s

po

2020

3-

419

34

17

26

10*99.12.82

10*20.2*8.0*6.1

10*2.82

10*20.2*8.0*1*10*6.1

10*20.210*2

)10*64.6(

VVoc 04.1)10*99.1

10*7ln(*0259.0

20

3

, as Isc>>Is

And )0259.0

1ln(*0259.004.1)*

1ln( mmm

V

kT

Vq

q

kTVocV

1- Assume Vm=0.9V LHS=0.947

2- Assume Vm=0.95V LHS=0.946

3- Assume Vm=0.945V LHS=0.946

Vm=0.946V

523

AmpI

IIsI

m

SCm

33152030.0259

0.946

20

kT

qV

10*85.610*710*28.7*10*99.110*7)1e(*10*99.1

)1e(m

Im(top)=6.85*10-3 Amp

Bottom cell calculations:

Same Isc in the bottom cell is given, we can calculate Voc.

In a practice, Isc may not be same as the top cell. We need to calculated it from power

incident from the top cell and power absorbed.

)ln(,

,,

,

bs

bsbSC

bOCI

II

q

kTV

From Q.1 of this set for crystalline Si cell, Is=0.24*10-12A~Is,b.

VVOC 624.0)10*0.24

10*7ln(0259.0)

10*0.24

10*0.2410*7ln(0259.0

12-

3

12-

-123

Now we calculate maximum power point Vmb Imb for bottom cell.

)0259.0

1ln(*0259.0624.0)*

1ln(,

mbmbBOCmb

V

kT

Vq

q

kTVV

Vmb=0.543V as shown in the table.

AI

III

mb

bSCbsmb

33330.0259

0.543

20

,kT

qV

,

10*69.610*710*305.010*7)1e(*10*99.1

)1e(mp

Imb(bottom cell)=6.69*10-3 A.

c) Tandem cell will be have lower of two current

I Tandem m = 6.69*10-3 A

V Tandem = Vm-top + Vm-bottom - 0.03

Vmp(total) =0.946 +0.543 -0.03=1.459 V

Total Power

= (Vmp-top+ Vmp-bottom -0.03) Imp-Top ,or = (Vmp-top+ Vmp-bottom -0.03) Imp-Bottom

=1.459*6.85*10-3

=9.994 mW

1- Assume Vm=0.6V LHS=0.541

2- Assume Vm=0.55V LHS=0.543

3- Assume Vm=0.544V LHS=0.543

524

6. 13 Derivation of V-I equation in an n-p solar cell

nr2=1.8

p-SinrSi=3.45nair=1

AR coating

t

F Photon

flux

X=0 X=t+d

Back contact

Front

contact

Distribution of electrons in the neutral p-region. In this treatment we assume the junction

width to be very small as compared with diffusion lengths and photon absorption lengths.

The continuity equation is expressed in terms of optical generation rate Gop, carrier

recombination rate R and the divergence term due to non-uniform current density.

RG

dx

xJd

qdt

dnop

n 1

, (78)

the generation rate depends on the magnitude of photon flux per unit area per second

incident on the n-Si face. The absorption coefficient (h) is a function of photon energy h.

The electron current density is due to diffusion and expressed as

dx

xndDqxJ nn

(79)

n

x

n

nFe

dx

xndD

dt

dn

2

2

(80)

Under steady state dn/dt =0,

n

x

n

xnFe

dx

xndD

)(0

2

2

22

2 )(

n

x

L

xnFe

dx

xnd

(81)

The solution is

1sinhcosh)(

22

2

nn

x

n

nn LD

eLF

L

txD

L

txCxn

(82)

The solution depends on boundary conditions. They are

BC #1

1kT

qV

po entxnxn

525

BC#2 0 dtxn

Substituting BC#1

tkT

qV

po TeDCen

0*1*1 , here

122

2

nn

n

LD

LFT

1

11

22

2

kT

qV

po

t

nn

nkT

qV

po

t eneLD

LFenTeC

(83)

Using BC#2

dt

nn

TeL

dD

L

dC

sinhcosh0

Substituting C from Eq. (83)

)(sinhcosh10 dt

nn

kT

qV

po

t TeL

dD

L

denTe

n

kT

qV

po

n

dt

n

L

den

L

deTe

L

dD cosh1cosh

sinh

1 (84)

Substituting C and D in n(x)

x

nn

kT

qV

po

n

dt

n

n

kT

qV

po

t

TeL

tx

L

den

L

deTe

L

d

L

txenTexn

sinhcosh1coshcosech

cosh1

(85)

The current density is x

xqDxJ n

nn

(86)

x

n

nn

kT

qV

po

n

dt

n

n

kT

qV

po

t

n

n

n

TeLL

tx

L

den

L

deTe

L

d

L

txenTe

L

DqxJ

coshcosh1coshcosech

sinh1

(87)

The current density at the junction boundary x = t

t

n

n

kT

qV

po

n

dt

nn

n

n TeLL

den

L

deTe

L

d

L

qDtxJ cosh1coshcosech

n

kT

qV

po

nn

d

n

t

n

n

nL

den

L

d

L

deLTe

L

qDtxJ coth1cothcosech (88)

Hole concentration in the window n-region: t +d > x > t, the continuity equation is

526

RG

dx

xJd

qdt

dpop

p

1 (89)

Under steady state, there is no time variation

p

xp xpeN

dx

xJd

q

1

0

N=Number of photon incidence per unit area per second= F

Under Steady State

p

xp xpeNq

dx

xJd

(90)

dx

pdDqxJ pp

, we neglect the drift term and also use F in place of N (91)

p

x

p

xpeFq

dx

pdqD

2

2

22

2

pp

x

L

xp

D

eF

dx

pd

(92)

x

pp

p

pp

eLD

LF

L

xB

L

xAxp

1sinhcosh)(

22

2

(93)

This can be written as

x

pp

eRL

xB

L

xAp

sinhcosh (94)

Where 122

2

pp

p

LD

FLR

(95)

The boundary conditions are

1

1# kT

qv

noeptxptxBC (96)

00,02# pqSJxBCand p (97)

Here, S is the surface recombination velocity at the x=0 interface.

Substituting the boundary condition BC#1 in p(x) expression at x= t

t

pp

kT

qV

no eRL

tB

L

tAep

sinhcosh1

tkT

qV

no

pp

eRepL

tB

L

tA

1sinhcosh (98)

Substituting the boundary condition BC#1 at x=0,

00*1* pJRBASq

0pJRqSASq (99)

We next find the Jp(0) expression.

527

x

p

ppp

p

x

pppp

p

pp

eRLL

xB

L

xA

L

qD

eRL

x

L

B

L

x

L

AqD

x

xpqDxJ

coshsin

coshsinh

)(

(100)

RqDBL

qDRLBA

L

qDJ p

p

p

p

p

p

p 0*)0( (101)

Substitute Eq. 101 Jp(0) in BC#1 Eq.99

Multiply by ‘-1’ and divide by qS

qS

RqD

qSL

BqD

qS

RSq

qS

ASq p

p

p

This gives

,S

RD

SL

BDRA

p

p

p

S

DR

SL

DBA

p

p

p 1 (102)

Substitute for A in Eq.98 to get an expression for B

Eq.3Re1sinhcosh tkT

qV

no

pp

p

p

peP

L

tB

L

t

S

DSR

LS

DB

Collect ‘B’ terms

tkT

qV

no

p

p

ppp

pep

L

t

S

DSR

L

t

L

t

LS

DB

Re1coshsinhcosh

ppp

p

tkT

qV

no

p

p

L

t

L

t

LS

D

epL

t

S

DSR

B

sinhcosh

Re1cosh

(103)

RqDBL

qDRSqASq p

p

p

528

Substitute B in Eq. 102 to get A

S

DR

L

t

L

t

LS

D

epL

t

S

DSR

LS

DA

p

ppp

p

tkT

qV

no

p

p

p

p

S

sinhcosh

Re1cosh

(104)

Reproduce equation 100

x

p

ppp

p

p LL

BL

AL

DqxJ Re

xcosh

xsinh

(100)

Jp at junction boundary x = t

t

p

ppp

p

p LL

BL

AL

DqtJ Re

tcosh

tsinh (105)

Substitute A from 104 & B from 103

Eq.7

sinhcosh

Re1cosh

*cosh

*sinhcosh

Re1cosh

*sinh

t

p

p

p

ppp

p

tkT

qV

no

p

p

pp

p

p

ppp

p

tkT

qV

no

p

p

p

p

pp

p

p

eRLL

Dq

L

t

L

t

LS

D

epL

t

S

DSR

L

t

L

Dq

S

DSR

L

t

L

t

LS

D

epL

t

S

DSR

LS

D

L

t

L

DqtJ

Cancelling Dp/SLp term from numerator and denominator in the first term,

t

pp

pp

p

pp

p

tkT

qV

no

p

p

pp

p

pp

p

p

tkT

qV

no

p

p

pp

p

p

eRLDq

L

t

D

LS

L

t

LS

D

epL

t

S

DR

L

t

L

Dq

L

t

D

LS

L

t

epL

t

S

DR

L

t

L

DqtJ

sinhcosh

Re1cosh1

*cosh

sinhcosh

Re1cosh1

*sinh

529

Separating pno(e

qV/kT -1) part from both first and second terms

pp

p

p

pp

p

pkT

qV

p

nop

p

L

t

D

LS

L

t

L

t

D

LS

L

t

eL

pDqtJ

sinhcosh

coshsinh

*1

t

p

pp

p

p

t

p

p

p

p

pp

t

p

p

p

peL

L

t

D

LS

L

t

eL

t

S

D

D

LS

L

t

L

te

L

t

S

D

L

DRq

sinhcosh

cosh1coshsinhcosh1

*

t

p

pp

p

p

pp

p

p

t

p

p

p

p

pp

p

p

pp

p

pkT

qV

p

nop

p

eL

L

t

D

LS

L

t

L

t

D

LS

L

te

L

t

S

D

L

DRq

L

t

D

LS

L

t

L

t

D

LS

L

t

eL

pDqtJ

sinhcosh

coshsinh*cosh1

*

sinhcosh

coshsinh

*1

(106)

Total current at the junction is obtained by adding Jp(t) from Eq. 106 and Jn(t) from Eq. 88.

t

p

pp

p

p

pp

p

p

t

p

p

p

p

pp

p

p

pp

p

pkT

qV

p

nop

np

eL

L

t

D

LS

L

t

L

t

D

LS

L

te

L

t

S

D

L

DRq

L

t

D

LS

L

t

L

t

D

LS

L

t

eL

pDq

txJtxJJ

sinhcosh

coshsinh*cosh1

*

sinhcosh

coshsinh

*1

n

kT

qV

po

nn

d

n

t

n

n

L

den

L

d

L

deLTe

L

qDcoth1cothcosech

here 122

2

pp

p

LD

FLR

or

122

p

p

p

p

L

FL

L

RD

, and

122

2

nn

n

LD

LFT

.or

122

n

n

n

n

L

LF

L

TD

Reorganizing the total current at the junction is

530

t

p

pp

p

p

pp

p

p

t

p

p

p

p

nn

pon

pp

p

p

pp

p

p

p

nopkT

qV

eL

L

t

D

LS

L

t

L

t

D

LS

L

te

L

t

S

D

L

LFq

L

d

L

nDq

L

t

D

LS

L

t

L

t

D

LS

L

t

L

pDqeJ

sinhcosh

coshsinh*cosh1

*1

coth

sinhcosh

coshsinh

**1

22

n

t

n

dtt

nn

n

L

de

L

deeL

L

LFq cothcosech

1

)(

22

(107)

The current can be written as

SC

kT

qV

S JeJJ 1 (108)

Where reverse saturation current density Js of a dark diode is

nn

pon

pp

p

p

pp

p

p

p

nop

SL

d

L

nDq

L

t

D

LS

L

t

L

t

D

LS

L

t

L

pDqJ coth

sinhcosh

coshsinh

* (109)

And photo-generated short circuit current density Jsc is

t

p

pp

p

p

pp

p

p

t

p

p

p

p

SC eL

L

t

D

LS

L

t

L

t

D

LS

L

te

L

t

S

D

L

LFqJ

sinhcosh

coshsinh*cosh1

*122

n

t

n

dtt

nn

n

L

de

L

deeL

L

LFq cothcosech

1

)(

22

(110)

The numerator in Jsc term

pp

p

p

t

p

p

L

t

D

LS

L

te

L

t

S

Dcoshsinh*cosh1

, simplifies to

=

pp

p

p

tp

pp

p

pL

t

D

LS

L

te

S

D

L

t

D

LSL coshsinh1

=

pp

p

p

t

p

p

pL

t

D

LS

L

te

D

LSL coshsinh , as

S

D

L

t p

p

1 is negligible.

Solar spectrum is covered if we sum q

eVh

Egh d

dp

hF

40 1q is the quantum efficiency.

531

6.13 Solar cell summary equations

Air mass m is definedzenithat is sun the whenlength path

tmeasuremen of timeat thesunlight theof length path opticalm .

Air mass m=2 or 1.5 is taken as a typical average for the day. At m=2, we have 74mW/cm2

which produces ISC = 27mA for a cell sample of 1cm2.

IL

ID

RS h

RS

I

RL

V-I Equation : Equivalent Circuit Approach

Generally, the behavior of a solar cell is modeled by the following equations. IL or ISC is the

current under short circuit condition generated by absorption of photons.

)I (orI-1)-e(I=I scLkT

qV

s (27)

Open circuit voltage is obtained by putting I=0 in Eq. 27 an rearranging terms.

I

I+I

q

kT=V

s

sLoc ln (36)

Maximum power point (Vm, Im) is obtained using I

V-=

I

V

, or

V

I-=

V

I

(40a, 40b) and Eq.

(27)

kT

Vq+1

q

kT-V=V

mocm ln (48)

Equation (48) can be solved numerically. Substituting Vm in Equation (27) results in an

expression for Im.

I-1]-e[I=I LkT

Vq

sm

m

(49)

Fill Factor: FF: The fill factor is related to the shape of the V-I plot.

rectangle outer of Area

rectangle inner of Area=

IV

P=

IV

IV=FF

Loc

m

Loc

mm

(50)

Photon energy required to generate an electron hole pair (EHP) = Eg= 1.1 eV in Si.

Excess energy per photon of energy = h (eV) – Eg = 1.5 1.1eV = 0.4eV (if hv = 1.5eV).

Excess energy not used to create electron hole pairs: If the power absorbed is Pabs1 and

532

average photon energy is hv, the excess energy

= Number of photon absorbed per sec x (hv – Eg).

Number of Photons absorbed / sec = sec/10*58.110*6.1*5.1

10*8.3 16

19

3

1 photonsh

Pabs

LOSSES: 1. Surface reflection:

Calculate reflectivity R = Reflectivity of Si = R = 3.01435.3

1435.322

airr

airr

nn

nn

Remedy: Design an antireflection coating using nr2 film

nr2=1.8

Si

nr=3.45nair=1

AR coating

t

Fig. 20 Antireflection coating. Fig. 32. Solar spectral irradiance for AM1.

)12(4 2

ln

tr

, photons of 1.5 eV, m

eVh

8266.0

5.1

24.124.1 )12(

8.1*4

10*8266.0 4

lt

,l =1,2

2. Long wavelength photons are not absorbed when hv < Eg. (as is very small.

3. Excess photon energy loss: Calculation (see design set page 414): Photon energy above

Eg is not used to generated electron-hole pairs (EHPs). This does not contribute to short

circuit current ISC or IL. Compute these losses: AM1 plot above h > 1.1eV

Regions are Triangle J, Trapezoid J, Trapezoid marked as #4, small rectangular regions

#5 & #6.

(triangle) J => Photon energy at = eV431.0

24.1

Photon energy at J = eV43.251.0

24.1 Average photon energy eVh ave 21.3

2

43.24

Excess photon energy not used in generating EHP= 3.21 – 1.1 = 2.11eV.

Area of the J = 2/155

2

31.051.0*1550

2* mW

JJ

Excess energy not used in triangle J is = 2/10111.2*

21.3

155mW

533

6.14 References 1. J. M. Olson and D. J. Friedman, High Efficiency iii-V Solar Cells, Compound

Semiconductor, Vol. 2, pp. 27-29, November /December 1996.

2. R. Metzger, Manufacturing III-V Solar Cells for Space Applications, Compound

Semiconductor, Vol. 2, pp. 25-26, November /December 1996.

3. M. Meyer and R. Metzger, Flying High: The Commercial Satellite Industry Converts to

Compound Semiconductor Solar Cells, Compound Semiconductor, Vol. 2, pp. 22-24,

November /December 1996.

4. K.A. Bertness, S. Kurtz, D.J. Friedman, A.E. Kibbler, C. Kramer, and J.M. Olson, 29.5%-

efficient GaInP/GaAs tandem solar cells, Appl. Phys. Lett., 65, pp. 989-991, August 1994.

5. N. Dharmarasu, M. Yamaguchi, and A. Khan, T. Yamada, T. Tanabe, S. Takagishi, T.

Takamoto, T. Ohshima, H. Itoh, M. Imaizumi and S. Matsuda, Appl. Phys. Lett., 79, pp. 2399-

2401, October 2001.

6. S. Licht, B. Wang, T. Soga, and M. Umeno, Light invariant, efficient, multiple band gap

AlGaAs/Si/metal hydride solar cell, Appl. Phys. Lett., 74, pp. 4055-4057, June 1999.

7. N. J. Ekins-Daukes, K.W. J. Barnham, J. P. Connolly, J.S. Roberts, J.C. Clark, and G. Hill,

Strain-balanced GaAsP/InGaAs quantum well solar cells, Appl. Phys. Lett., 75, pp. 4195-4197,

December 1999.

8. D. W. Parent, A. Rodriguez, J. E. Ayers, and F. C. Jain, Photoassisted MOVPE Grown (n)

ZnSe/(p+)GaAS heterojunction solar cells, Solid-State Electronics, 47, pp. 595-599, 2003.

9. F. C. Jain, Observation of an active photoinductive component in high intensity solar cells.

J. App. Phys. 51, p. 2685, 2000.

10. S. M. Sze, Physics of Semiconductors, Wiley-Interscience, New York, 1981.

11. A.C. Miles, Semiconductor Devices and Integrated Circuits, Van Nostrand, Reinhold, p.736

1980.

12. R.A Metzger, Manufacturing III-V Solar cells for space Application, Page 25,

Compound Semiconductor November / December 1996.

13. Mary Archer and Bob Hill, Eds., Clean Energy from Photovoltaics (1st Edition), Series on

Photoconversion of Solar Energy – Vol. 1, (Imperial College Press, London, 2001).[covers

Amorphous Silicon Solar Cells by C. Wronski and D. Carlson, Cadmium Telluride Solar

Cells –by D. Bonnet, Cu(In,Ga)Se- Solar Cells by U. Rau and H. Schock.

14. A. Luque and S. Hegedus, Eds. Handbook of Photovoltaics Science and Engineering

(Wiley, West Sussex, 2003), Cadmium Telluride Solar Cells by B.E. McCandless and J. R.

Sites.

15. J. Poortmans and V. Arkhipov, Eds., Thin Film Solar Cells: Fabrication, Characterization,

and Applications (Wiley, West Sussex, 2006). [covers Chalcopyrite Based Solar Cells. R.

Klenk and M. C. Lux-Stiner, and Cadmium Telluride Solar Cells byMarc Burgelman.]

16, Lewis Fraas and Larry Partain, Eds., Solar Cells and Their Applications, 2nd Edition,

(Wiley, New Jersey, 2010). See Thin-Film Solar Cells and Modules by Robert Birkmier.

17.W. van Sark, Ed. Comprehensive Renewable Energy, Volume 1: Photovoltaic Solar Energy

(Elsevier, 2012).[covers Thin-Film Silicon PV Technology by R.E.I. Schorpp, Chalcopyrite

Thin Film Materials and Solar Cells byT. Unold and C.A. Kaufmann, and Cadmium

Telluride Photovoltaic Thin Film Solar Cells by T.A. Gessert.

18. A.J. Nozik, M.C. Beard, and G. Conibeer, Eds. Advanced Concepts in Photovoltaics,

Royal Society of Chemistry (RSC) Publishing, London, 2014. See “Thin-Film CdTe

Photovoltaic Solar Cell Devices,” by T.A. Gessert, B. McCandless, and C. Ferekides.

19.Mary Archer and Martin Greene, Eds. Clean Energy from Photovoltaics, 2nd Edition, Series

on Photoconversion of Solar Energy – Vol. 1, World Series Publishing, London, 2014). See

534

Polycrystalline Cadmium Telluride Photovoltaic Devices by T.A. Gessert and D. Bonnet.

20. J. Perrenoud, S. Buecheler, A.N. Tiwari, “Flexible CdTe Solar Cells with High

Photovoltaic Conversion Efficiency”, 2009 IEEE Conference, p.695-699, 2009.

21. A. M. Green , K. Emery , Y. Hishikawa , and W. Warta . Prog. Photovoltaics 17 , 85 – 94 ( 2009).